NCLEX Coronary Vascular Disorders

Réussis tes devoirs et examens dès maintenant avec Quizwiz!

During a teaching session, a client who is having a valuloplasty tomorrow asks the nurse about the difference between a mechanical valve replacement and a tissue valve. Which of the following answers from the nurse is correct information?

"A mechanical valve is thought to be more durable and so requires replacement less often." Explanation: Mechanical valves are thought to be more durable than tissue valves and so require replacement less often. Tissue valves are less likely to generate blood clots and so long-term anticoagulation therapy is not required. Homografts (human valves) are not always available and are very expensive.

A nurse is teaching a client who is having a valuloplasty tomorrow. The client asks what the advantage is for having a tissue valve replacement instead of a mechanical valve. The correct answer by the nurse is which of the following?

"A tissue valve is less likely to generate blood clots, and so long-term anticoagulation therapy is not required." Explanation: Tissue valves are less likely to generate thromoemboli, so long-term anticoagulation is not required. Mechanical valves do not deteriorate or become infected as easily as tissue valves. They are thought to be more durable than tissue valves and so require replacement less often

The client asks the nurse to explain the difference between arteriosclerosis and atherosclerosis. Which is the best explanation provided by the nurse?

"Arteriosclerosis is a condition that produces structural changes in the arteries, and atherosclerosis is a specific type of arteriosclerosis." Explanation: Arteriosclerosis is a complex condition that produces structural changes to the arteries usually associated with loss of elasticity. Atherosclerosis is a specific type and most common cause of arteriosclerosis. Both disorders affect the ability of the vessels to deliver blood and are considered occlusive disorders, but the causes differ. Vessels that become dilated and weakened are referred to as aneurysms, not arteriosclerosis.

A nurse and a nursing student are caring for a client with pericarditis and perform the physical assessment together. The client has a pericardial friction rub audible on auscultation. When the nurse and student leave the room, the student asks how to distinguish a pericardial from a pleural friction rub. The nurse's best response is which of the following?

"Ask the client to hold the breath while you auscultate; the pericardial friction rub will continue, while the pleural friction rub will stop." Explanation: A pericardial friction rub occurs when the pericardial surfaces lose their lubricating fluid as a result of inflammation. The audible rub on auscultation is synchronous with the heartbeat. To distinguish between a pleural rub and a pericardial rub, the client should hold the breath. The pericardial rub will continue.

A client with known coronary artery disease reports intermittent chest pain, usually on exertion. The physician diagnoses angina pectoris and orders sublingual nitroglycerin to treat acute angina episodes. When teaching the client about nitroglycerin administration, which instruction should the nurse provide?

"Be sure to take safety precautions because nitroglycerin may cause dizziness when you stand up." Nitroglycerin commonly causes orthostatic hypotension and dizziness. To minimize these problems, the nurse should teach the client to take safety precautions, such as changing to an upright position slowly, climbing up and down stairs carefully, and lying down at the first sign of dizziness. To ensure the freshness of sublingual nitroglycerin, the client should replace tablets every 6 months, not every 9 months, and store them in a tightly closed container in a cool, dark place. Many brands of sublingual nitroglycerin no longer produce a burning sensation. The client should take a sublingual nitroglycerin tablet at the first sign of angina. He may repeat the dose every 5 minutes for up to three doses; if this intervention doesn't bring relief, the client should seek immediate medical attention.

A client comes to the physician's office for a follow-up visit 4 weeks after suffering a myocardial infarction (MI). The nurse takes this opportunity to evaluate the client's knowledge of the ordered cardiac rehabilitation program. Which evaluation statement suggests that the client needs more instruction?

"Client walks 4 miles in 1 hour every day." Explanation: Four weeks after an MI, a client's walking program should aim for a goal of 2 miles in less than 1 hour. Walking 4 miles in 1 hour is excessive and may induce another MI by increasing the heart's oxygen demands. Therefore, this client requires appropriate exercise guidelines and precautions. Performing relaxation exercises; following a low-fat, low-cholesterol diet; and seeking emergency help if the heart rate increases markedly at rest indicate understanding of the cardiac rehabilitation program. For example, the client should reduce stress, which speeds the heart rate and thus increases myocardial oxygen demands. Reducing dietary fat and cholesterol intake helps lower risk of atherosclerosis. A sudden rise in the heart rate while at rest warrants emergency medical attention because it may signal a life-threatening arrhythmia and increase myocardial oxygen demands.

A client with angina pectoris must learn how to reduce risk factors that exacerbate this condition. When developing the client's care plan, which expected outcome should a nurse include?

"Client will verbalize the intention to stop smoking." A client with angina pectoris should stop smoking at once because smoking increases the blood carboxyhemoglobin level; this increase, in turn, reduces the heart's oxygen supply and may induce angina.

A nurse is caring for a client who has hypertension and diabetes mellitus. The client's blood pressure this morning was 150/92 mm Hg. He asks the nurse what his blood pressure should be. The nurse's most appropriate response is:

"Clients with diabetes should have a lower blood pressure goal. You should strive for 130/80 mm Hg."

A client with a strong family history of coronary artery disease asks the nurse how to reduce the risk of developing the disorder. Which is the best response by the nurse?

"Exercise, keep your blood sugar in check, and manage your stress." Explanation: Although moderation is the key, this does not provide specific options for this client such as regular exercise and managing stress and cholesterol levels. The reverse lipid drug sounds good but is not available or approved by the FDA. Soy products have limited benefits for cholesterol control.

The nurse determines that a 49-year-old client recently diagnosed with subacute bacterial endocarditis understands discharge teaching upon which client statement?

"I have to call my doctor so I can get antibiotics before seeing the dentist." Explanation: Antibiotic prophylaxis is recommended for high-risk clients immediately before and sometimes after dental procedures.

A client, who has undergone a percutaneous transluminal coronary angioplasty (PTCA), has received discharge instructions. Which statement by the client would indicate the need for further teaching by the nurse?

"I should expect a low-grade fever and swelling at the site for the next week." Fever and swelling at the site are signs of infection and should be reported to the physician. Showers should be taken until the insertion site is healed. Prolonged sitting can result in thrombosis formation. Bruising at the insertion site is common and may take from 1 to 3 weeks to resolve.

A nurse is caring for a client with heart failure. The nurse knows that the client has left-sided heart failure when he makes which statement?

"I sleep on three pillows each night"

A client with an acute myocardial infarction is receiving nitroglycerin by continuous I.V. infusion. Which client statement indicates that this drug is producing its therapeutic effect?

"My chest pain is decreasing." Explanation: Nitroglycerin, a vasodilator, increases the arterial supply of oxygen-rich blood to the myocardium. This action produces the drug's intended effect: relief of chest pain. Headache is an adverse effect of nitroglycerin. The drug shouldn't cause a tingling sensation around the mouth and should lower, not raise, blood pressure

A nurse is caring for a client who is recovering from a myocardial infarction (MI). The cardiologist refers him to cardiac rehabilitation. Which statement by the client indicates an understanding of cardiac rehabilitation?

"Rehabilitation will help me function as well as I physically can." Explanation: Reference: Hinkle, J.L., & Cheever, K.H., Brunner & Suddarth's Textbook of Medical-Surgical Nursing, 14th ed., Philadelphia, Wolters Kluwer, 2018, Chapter 27: Management of Patients With Coronary Vascular Disorders, Cardiac Rehabilitation, p. 766.

A client is ordered a nitroglycerine transdermal patch for treatment of CAD and asks the nurse why the patch is removed at bedtime. Which is the best response by the nurse?

"Removing the patch at night prevents drug tolerance while keeping the benefits." Tolerance to antiangina effects of nitrates can occur when taking these drugs for long periods of time. Therefore, to prevent tolerance and maintain benefits, it is a common regime to remove transdermal patches at night. Common adverse effects of nitroglycerine are headaches and contact dermatitis but not the reason for removing the patch at night. It is true that while you rest, there is less demand on the heart but not the primary reason for removing the patch.

When assessing a client who reports recent chest pain, the nurse obtains a thorough history. Which client statement most strongly suggests angina pectoris?

"The pain occurred while I was mowing the lawn." Decreased oxygen supply to the myocardium causes angina pectoris. Lawn mowing increases the cardiac workload, which increases the heart's need for oxygen and may precipitate this chest pain. Anginal pain typically is self-limiting, lasting 5 to 15 minutes. Food consumption doesn't reduce angina pain, although it may ease pain caused by a GI ulcer. Deep breathing has no effect on anginal pain.

A nursing student is assigned to a patient with a mechanical valve replacement. The patient asks the student, "Why do I have to take antibiotics before getting my teeth cleaned?" Which response by the nursing student is most appropriate?

"You are at risk of developing an infection in your heart." Explanation: The patient is at risk for endocarditis and should take prophylactic antibiotics before any invasive procedure. Antibiotics have nothing to do with how much the teeth bleed. Cleaning of the teeth will not cause the valve to malfunction. Using the word "vegetative" with the patient may not be understood; therefore, it is not the most appropriate answer.

A man has just arrived in the ER with a possible myocardial infarction (MI). The electrocardiogram (ECG) should be obtained within which time frame of arrival to the ER?

10 minutes Explanation: The ECG provides information that assists in diagnosing acute MI. It should be obtained within 10 minutes from the time a patient reports pain or arrives in the emergency department. By monitoring serial ECG changes over time, the location, evolution, and resolution of an MI can be identified and monitored.

A client presents to the emergency department reporting chest pain. Which order should the nurse complete first?

12 lead ECG

When monitoring a patient who has hypertension and chronic kidney disease, the target pressure for this individual should be less than which blood pressure reading?

130/80 mmHg

Protein should make up approximately _____ of total calories.

15%

The nurse is reviewing the results of a total cholesterol level for a patient who has been taking simvastatin (Zocor). What results display the effectiveness of the medication?

160-190 mg/dL Simvastatin (Zocor) is a statin Frequently given as initial therapy for significantly elevated cholesterol and low-density lipoprotein levels. Normal total cholesterol is less than 200 mg/dL

Heparin therapy is usually considered therapeutic when the patient's activated partial thromboplastin time (aPTT) is how many times normal?

2 to 2.5 The amount of heparin administered is based on aPTT results, which should be obtained in follow-up to any alteration of dosage. The patient's aPTT value would have to be greater than .5 to 1 times normal to be considered therapeutic. An aPTT value that is 2.5 to 3 times normal would be too high to be considered therapeutic. The patient's aPTT value would have to be greater than .25 to .75 times normal to be considered therapeutic.

What is the normal CVP level?

2-6

Dietary fiber should be ___to___ grams per day.

20-30

A client is receiving intravenous heparin to prevent blood clots. The order is for heparin 1,200 units per hour. The pharmacy sends 25,000 units of heparin in 500 mL of D5W. At how many mL per hour will the nurse infuse this solution? Enter the correct number ONLY.

24 ml

A patient asks the nurse how long he will have to wait after taking nitroglycerin before experiencing pain relief. What is the best answer by the nurse?

3 minutes Nitroglycerin may be given by several routes: sublingual tablet or spray, oral capsule, topical agent, and intravenous (IV) administration. Sublingual nitroglycerin is generally placed under the tongue or in the cheek (buccal pouch) and ideally alleviates the pain of ischemia within 3 minutes.

The nurse is explaining the DASH diet to a patient diagnosed with hypertension. The patients inquires about how many servings of fruit per day can be consumed on the diet. The nurse would be correct in stating which of the following?

4 to 5

A client with suspected acute myocardial infarction is admitted to the coronary care unit. To help confirm the diagnosis, the physician orders serial enzyme tests. Increased serum levels of the isoenzyme creatinine kinase of myocardial muscle (CK-MB), found only in cardiac muscle, can be detected how soon after the onset of chest pain?

4 to 6 hours Serum CK-MB levels can be detected 4 to 6 hours after the onset of chest pain. These levels peak within 12 to 18 hours and return to normal within 3 to 4 days.

Following a percutaneous transluminal coronary angioplasty, a client is monitored in the postprocedure unit. The client's heparin infusion was stopped 2 hours earlier. There is no evidence of bleeding or hematoma at the insertion site, and the pressure device is removed. The nurse should plan to remove the femoral sheath when the partial thromboplastin time (PTT) is:

50 seconds or less. Heparin causes an elevation of the PTT and, thereby, increases the risk for bleeding. With a large cannulation such as a sheath used for angioplasty, the PTT should be 50 seconds or less before the sheath is removed. Removing the sheath before the PTT drops below 50 seconds can cause bleeding at the insertion site. The other PTT results are incorrect for determining when to remove the sheath.

Carbohydrates should make up ____% of total calories

50-60%

In order to be effective, percutaneous transluminal coronary angioplasty (PTCA) must be performed within what time frame, beginning with arrival at the emergency department after diagnosis of myocardial infarction?

60 minutes

To be effective, percutaneous transluminal coronary angioplasty (PTCA) must be performed within what time frame, beginning with arrival at the emergency department after diagnosis of myocardial infarction (MI)?

60 minutes

What is myoglobin?

A biomarker that rises in 2 to 3 hours after heart damage during an MI

The nurse has been asked to explain the cause of angina pain to a patient's family. Choose the best statement. The pain is due to:

A lack of oxygen in the heart muscle that causes the death of cells. Impeded blood flow, due to blockage in a coronary artery, deprives the cardiac muscle cells of oxygen thus leading to a condition known as ischemia.

Post-cardiac surgery assessment of renal function should be performed hourly for the first 12 to 24 hours. Identify the laboratory result that the nurse knows is a primary indicator of possible renal failure.

A serum BUN of 70 mg/dL These four laboratory results should always be assessed, post cardiac surgery. Serum osmolality (N = >800 mOsm/kg) should also be included. A BUN reading of greater than 21 mg/dL is abnormal; a reading of greater than 60 mg/dL is indicative of renal failure. The lab results in the other choices are all within normal range

The nurse is assessing a postoperative patient who had a percutaneous transluminal coronary angioplasty (PTCA). Which possible complications should the nurse monitor for?

Abrupt closure of the artery Arterial dissection Coronary artery vasospasm

Which nursing diagnosis is most significant in planning the care for a client with Raynaud's disease?

Acute Pain The hallmark symptom of Raynaud's Disease is pain related to the arterial insufficiency. Disturbed Sensory Perception associated with paresthesia can occur but is less significant than pain. Self-Care Deficit and Activity Intolerance can occur but less significant than Acute Pain.

Shortly after being admitted to the coronary care unit with an acute myocardial infarction (MI), a client reports midsternal chest pain radiating down his left arm. The nurse notes that the client is restless and slightly diaphoretic, and measures a temperature of 99.6° F (37.6° C); a heart rate of 102 beats/minute; regular, slightly labored respirations at 26 breaths/minute; and a blood pressure of 150/90 mm Hg. Which nursing diagnosis takes highest priority?

Acute pain Explanation: The nursing diagnosis of Acute pain takes highest priority because it increases the client's pulse and blood pressure. During the acute phase of an MI, low-grade fever is an expected result of the body's response to myocardial tissue necrosis. This makes Risk for imbalanced body temperature an incorrect answer. The client's blood pressure and heart rate don't suggest a nursing diagnosis of Decreased cardiac output. Anxiety could be an appropriate nursing diagnosis, but addressing Acute pain (the priority concern) may alleviate the client's anxiety.

A client with left-sided heart failure complains of increasing shortness of breath and is agitated and coughing up pink-tinged, foamy sputum. The nurse should recognize these findings as signs and symptoms of:

Acute pulmonary edema

The charge nurse was discussing with the nursing student that studies have been published that suggest inflammation increases the risk of heart disease. Which modifiable factor would the nursing student target in teaching clients about prevention of inflammation that can lead to atherosclerosis?

Addressing obesity Explanation: Published information by Balistreri et al. (2010) indicated a relationship between body fat and the production of inflammatory and thrombotic (clot-facilitating) proteins. This information suggests decreasing obesity and body fat stores may help to reduce the risk. Avoiding the use of caffeine, encouraging the use of a multivitamin, and drinking at least 2 liters of water a day are not actions that will address the prevention of inflammation that can lead to artherosclerosis.

A client in the emergency department complains of squeezing substernal pain that radiates to the left shoulder and jaw. He also complains of nausea, diaphoresis, and shortness of breath. What should the nurse do?

Administer oxygen, attach a cardiac monitor, take vital signs, and administer sublingual nitroglycerin. Cardiac chest pain is caused by myocardial ischemia. Therefore the nurse should administer supplemental oxygen to increase the myocardial oxygen supply, attach a cardiac monitor to help detect life-threatening arrhythmias, and take vital signs to ensure that the client isn't hypotensive before giving sublingual nitroglycerin for chest pain. Registration information may be delayed until the client is stabilized. Alerting the cardiac catheterization team or the physician before completing the initial assessment is premature.

A nurse is reevaluating a client receiving IV fibrinolytic therapy. Which of the following patient findings requires immediate intervention by the nurse?

Altered level of consciousness A patient receiving fibrinolytic therapy is at risk for complications associated with bleeding. Altered level of consciousness may indicate hypoxia and intracranial bleeding and the infusion should be discontinued immediately. Minimal bleeding requires manual pressure. Reperfusion dysrhythmias are an expected finding. A chest pain score of 2 is low, and indicates the patient's chest pain is subsiding, an expected outcome of this therapy.

A nurse plans to have an education session with a client with cardiomyopathy and the client's spouse about ways to increase activity tolerance. Which of the following instructions would provide that information?

Alternate active periods with rest periods. Explanation: The client should plan activities to occur in cycles, alternating rest with active periods. The client with cardiomyopathy must avoid strenuous activity and isometric exercises. It is impossible to avoid all physical and emotional stress.

What ECG findings does the nurse observe in a patient who has had a myocardial infarction (MI)? (Select all that apply.)

An abnormal Q wave T-wave inversion ST-segment elevation Explanation: The ECG changes that occur with an MI are seen in the leads that view the involved surface of the heart. The expected ECG changes are T-wave inversion, ST-segment elevation, and development of an abnormal Q wave (Fig. 27-5).

What ECG findings does the nurse observe in a patient who has had a myocardial infarction (MI)? (Select all that apply.)

An abnormal Q wave T-wave inversion ST-segment elevation The ECG changes that occur with an MI are seen in the leads that view the involved surface of the heart. The expected ECG changes are T-wave inversion, ST-segment elevation, and development of an abnormal Q wave (Fig. 27-5).

The nurse is auscultating the heart of a patient diagnosed with mitral valve prolapse. Which of the following is often the first and only manifestation of mitral valve prolapse?

An extra heart sound Explanation: Often, the first and only sign of mitral valve prolapse is identified when a physical examination of the heart reveals an extra heart sound referred to as a mitral click. Fatigue, dizziness, and syncope are other symptoms of mitral valve prolapsed.

Which of the following terms refers to chest pain brought on by physical or emotional stress and relieved by rest or medication?

Angina pectoris Angina pectoris is a symptom of myocardial ischemia. Atherosclerosis is an abnormal accumulation of lipid deposits and fibrous tissue within arterial walls and lumens. Atheromas are fibrous caps composed of smooth muscle cells that form over lipid deposits within arterial vessels. Ischemia is insufficient tissue oxygenation and may occur in any part of the body.

Which of the following statements is not accurate regarding an autograft?

Anticoagulation is necessary

Following a percutaneous transluminal coronary angioplasty (PTCA), which of the following medications classifications would be used to prevent thrombus formation in the stent?

Antiplatelets Explanation: Because of the risk of thrombus formation following a coronary stent placement, the patient receives antiplatelet medications, such as Plavix or aspirin. Nitrates, beta blockers, and calcium channel blockers would not be used for this purpose.

Which of the following methods to induce hemostasis after sheath removal post percutaneous transluminal coronary angioplasty (PTCA) is the least effective?

Application of a sandbag to the area Several nursing interventions frequently used as part of the standard of care, such as applying a sandbag to the sheath insertion site, have not been shown to be effective in reducing the incidence of bleeding. Application of a vascular closure device has been demonstrated to be very effective. Direct manual pressure to the sheath introduction site has been demonstrated to be effective and was the first method used to induce hemostasis post PTCA. Application of a pneumatic compression device post PTCA has been demonstrated to be effective.

After percutaneous transluminal coronary angioplasty (PTCA), the nurse suspects that a patient, who is on bed rest, may be experiencing the complication of bleeding. The nurse's initial action should be to do what?

Apply manual pressure at the site of the insertion of the sheath.

After percutaneous transluminal coronary angioplasty (PTCA), the nurse suspects that a patient, who is on bed rest, may be experiencing the complication of bleeding. The nurse's initial action should be to do which of the following?

Apply manual pressure at the site of the insertion of the sheath. The immediate nursing action would be to apply pressure, which may stop the bleeding. If the bleeding does not stop, the health care provider needs to be notified.

A client asks the clinic nurse what the difference is between arteriosclerosis and atherosclerosis. What is the nurse's best response?

Arteriosclerosis is a loss of elasticity, or hardening of the arteries, that happens as we age Arteriosclerosis refers to the loss of elasticity or hardening of the arteries, that accompanies the aging process. Therefore, options B, C, and D are incorrect.

While assessing a patient with pericarditis, the nurse cannot auscultate a friction rub. Which action should the nurse implement?

Ask the patient to lean forward and listen again. Explanation: The most characteristic sign of pericarditis is a creaky or scratchy friction rub heard most clearly at the left lower sternal border. Having the patient lean forward and to the left uses gravity to force the heart nearer to the chest wall, which allows the friction rub to be heard. These assessment data are not life-threatening and do not require a call to the health care provider. The nurse should try multiple times to auscultate the friction rub before deciding that the rub is gone. Chest tubes are not the treatment of choice for not hearing friction rubs.

A patient complains about chest pain and heavy breathing when exercising or when stressed. Which of the following is a priority nursing intervention for the patient diagnosed with coronary artery disease?

Assess chest pain and administer prescribed drugs and oxygen The nurse assesses the patient for chest pain and administers the prescribed drugs that dilate the coronary arteries. The nurse administers oxygen to improve the oxygen supply to the heart. Assessing the blood pressure or the physical history does not clearly indicate that the patient has CAD. The nurse does not administer aspirin without the physician's prescription.

You are caring for a client with CAD. What is an appropriate nursing action when evaluating a client with coronary artery disease (CAD)?

Assess the characteristics of chest pain. Rationale: The nurse should assess the characteristics of chest pain for a client with CAD. Assessing the client's mental, emotional status, the skin of the client, or for drug abuse will not assist the nurse in evaluating the client for CAD.

You are caring for a client with coronary artery disease (CAD). What is an appropriate nursing action when evaluating a client with CAD?

Assess the characteristics of chest pain. The nurse should assess the characteristics of chest pain for a client with CAD. Assessing the client's mental and emotional status, skin, or for drug abuse will not assist the nurse in evaluating the client for CAD. The assessment should be aimed at evaluating for adequate blood flow to the heart.

A nurse is caring for a client who is exhibiting signs and symptoms characteristic of a myocardial infarction (MI). Which statement describes priorities the nurse should establish while performing the physical assessment?

Assess the client's level of pain and administer prescribed analgesics. The cardinal symptom of MI is persistent, crushing substernal pain or pressure. The nurse should first assess the client's pain and prepare to administer nitroglycerin or morphine for pain control. The client must be medically stabilized before pulmonary artery catheterization can be used as a diagnostic procedure.

A nurse plans to have an education session with a client who has cardiomyopathy and the client's spouse about ways to decrease the sense of powerlessness. Which of the following actions by the nurse will provide this information?

Assist the client in identifying life areas over which she still has control. Explanation: The client should be assisted to identify the things in life that she has lost. The nurse will then assist the client to identify the amount of control that she still has (eg, low-sodium food choices, medication schedule). The nurse should never encourage the client to forget what she has lost or change the subject if a client wants to talk about something. The nurse should encourage this discussion. It is important that the client adhere closely to a low-sodium diet. The nurse should encourage this adherence and never suggest otherwise.

The nurse plays an important role in monitoring and managing potential complications in the patient who has recently undergone a coronary artery bypass graft (CABG). The nurse should be alert to which of the following respiratory complications?

Atelectasis Respiratory complications that may occur include atelectasis. An incentive spirometer and the use of deep breathing exercises are necessary to prevent atelectasis and pneumonia. Elevated blood sugar levels, hyperkalemia, UTI, and are complications that can occur but are unrelated to the respiratory system.

Which condition most commonly results in coronary artery disease (CAD)?

Atherosclerosis Atherosclerosis (plaque formation), is the leading cause of CAD. Diabetes mellitus is a risk factor for CAD, but it isn't the most common cause. Myocardial infarction is a common result of CAD. Renal failure doesn't cause CAD, but the two conditions are related.

You are presenting a workshop at the senior citizens center about how the changes of aging predispose clients to vascular occlusive disorders. What would you name as the most common cause of peripheral arterial problems in the older adult?

Atherosclerosis Atherosclerosis is the most common cause of peripheral arterial problems in the older adult. The disease correlates with the aging process. The other choices may occur at any age.

When starting a client on oral or I.V. diltiazem (Cardizem), for which potential complication should the nurse monitor?

Atrioventricular block Explanation: The chief complications of diltiazem are hypotension, atrioventricular blocks, heart failure, and elevated liver enzyme levels. Other reported reactions include flushing, nocturia, and polyuria, but not renal failure. Although flushing may occur, it's an adverse reaction, not a potential complication.

Which medication should a nurse have on hand when removing a sheath after cardiac catheterization?

Atropine Explanation: Removing the sheath after cardiac catheterization may cause a vasovagal response, including bradycardia. The nurse should have atropine on hand to increase the client's heart rate if this occurs. Heparin thins the blood; clients should stop taking it before the sheath removal. Protamine sulfate is an antidote to heparin, but the nurse shouldn't administer it during sheath removal. Adenosine treats tachyarrhythmias.

The nurse administers propranolol hydrochloride to a patient with a heart rate of 64 beats per minute (bpm). One hour later, the nurse observes the heart rate on the monitor to be 36 bpm. What medication should the nurse prepare to administer that is an antidote for the propranolol?

Atropine Sheath removal and the application of pressure on the vessel insertion site may cause the heart rate to slow and the blood pressure to decrease (vasovagal response). A dose of IV atropine is usually given to treat this response.

Which of the following describes a valve used in replacement surgery that is made from the patient's own heart valve?

Autograft. Explanation: An example of autograft is found when the surgeon excises the pulmonic valve and uses it for an aortic valve replacement. Allograft refers to replacement using human tissue and is a synonym for homograft. Homograft refers to replacement using human tissue and is a synonym for allograft. Xenograft refers to replacement of tissue from animal tissue.

The nurse is caring for a client with Raynaud's disease. What is an important instruction for a client who is diagnosed with this disease to prevent an attack?

Avoid situations that contribute to ischemic episodes.

Which of the following is a key diagnostic indicator of heart failure (HF)?

BNP

The nurse is caring for a client who is diagnosed with aortic stenosis, but is reluctant to have surgery for valve replacement. The nurse is present when the physician talks to the client about a treatment that is less invasive than surgery, but that will likely relieve some of the client's symptoms. Later, the client asks the nurse to review with her spouse the treatment offered. What does the nurse talk about with the spouse?

Balloon percutaneous valvuloplasty Explanation: Definitive treatment for aortic stenosis is surgical replacement of the aortic valve. Clients who are symptomatic, but not good surgical candidates may benefit from a one or two balloon percutaneous valvuloplasty. Antibiotic therapy will not open the valve. The client does not want to have a valve replacement of any kind.

Aortic stenosis remains asymptomatic for several decades. However, once a client becomes symptomatic for aortic stenosis, life expectancy without further treatment is only 2 to 3 years. What is the treatment of choice for symptomatic aortic stenosis?

Balloon valvuloplasty Additional treatment eventually becomes critical because average survival is 2 to 3 years once symptoms develop. Balloon valvuloplasty is an invasive, nonsurgical procedure to enlarge a narrowed valve opening. Balloon angioplasty, cardiac catheterization, and cardiac graft procedure are not indicated treatments for aortic stenosis.

A client is returning from the operating room after inguinal hernia repair. The nurse notes that he has fluid volume excess from the operation and is at risk for left-sided heart failure. Which sign or symptom indicates left-sided heart failure?

Bibasilar crackles

A client is receiving nitroglycerin ointment (Nitro-Dur) to treat angina pectoris. The nurse evaluates the therapeutic effectiveness of this drug by assessing the client's response and checking for adverse effects. Which vital sign is most likely to reflect an adverse effect of nitroglycerin?

Blood pressure 84/52 mm Hg Explanation: Hypotension and headache are the most common adverse effects of nitroglycerin. Therefore, blood pressure is the vital sign most likely to reflect an adverse effect of this drug. The nurse should check the client's blood pressure 1 hour after administering nitroglycerin ointment. A blood pressure decrease of 10 mm Hg is within the therapeutic range. If blood pressure falls more than 20 mm Hg below baseline, the nurse should remove the ointment and report the finding to the physician immediately. An above-normal heart rate (tachycardia) is a less common adverse effect of nitroglycerin. Respiratory rate and temperature don't change significantly after nitroglycerin administration.

Which diagnostic is a marker for inflammation of vascular endothelium?

C-reactive protein (CRP) C-reactive protein (CRP) is a marker for inflammation of vascular endothelium. LDL, HDL, and triglycerides are not marker of vascular endothelium inflammation. They are elements of fat metabolism.

A nurse is caring for a patient in the cardiovascular intensive care unit (CVICU) following a coronary artery bypass graft (CABG). Which of the following clinical findings requires immediate intervention by the nurse?

CVP reading: 1 mmHg The central venous pressure (CVP) reading of 1 is low (2-6 mmHg) and indicates reduced right ventricular preload, commonly caused by hypovolemia. Hypovolemia is the most common cause of decreased cardiac output after cardiac surgery. Replacement fluids such as colloids, packed red blood cells, or crystalloid solutions may be prescribed. The other findings require follow-up by the nurse; however, addressing the CVP reading is the nurse's priority.

A 56-year-old male client with CAD kept his appointment to see a cardiologist for posthospitalization follow up. The client indicates that he thinks the diltiazem (Cardizem) has been making him nauseous. The nurse is aware that diltiazem (Cardizem) is categorized as which type of drug?

Calcium-channel blockers

A client kept his appointment to see a cardiologist for post-hospitalization follow up. The cardiologist examines him and inquires as to the medication's efficacy since his hospitalization. Effective medications are intended to produce arterial vasodilation. What type of medication, listed below, is not as an effective vasodilator as others?

Calcium-channel blockers Explanation: Calcium-channel blocking agents may be used to treat CAD as well, although research has shown that they may be less beneficial than beta-adrenergic blocking agents.

LDH1 and LDH2 may be elevated in response to...

Cardiac or other organ damage during an MI.

A patient has had cardiac surgery and is being monitored in the intensive care unit (ICU). What complication should the nurse monitor for that is associated with an alteration in preload?

Cardiac tamponade Preload alterations occur when too little blood volume returns to the heart as a result of persistent bleeding and hypovolemia. Excessive postoperative bleeding can lead to decreased intravascular volume, hypotension, and low cardiac output. Bleeding problems are common after cardiac surgery because of the effects of cardiopulmonary bypass, trauma from the surgery, and anticoagulation.

Which complication of cardiac surgery occurs when there is fluid and clot accumulation in the pericardial sac, which compresses the heart, preventing blood from filling the ventricles?

Cardiac tamponade Rationale: Cardiac tamponade is fluid and clot accumulation in the pericardial sac, which compresses the heart, preventing the blood from filling the ventricles. Fluid overload is exhibited by high PAWP, CVP, and pulmonary artery diastolic pressure as well as crackles in the lungs. Hypertension results from postoperative vasoconstriction. Hypothermia is a low body temperature that leads to vasoconstriction.

Which pulse should the nurse palpate during rapid assessment of an unconscious adult?

Carotid

A client with chest pain arrives in the emergency department and receives nitroglycerin, morphine (Duramorph), oxygen, and aspirin. The physician diagnoses acute coronary syndrome. When the client arrives on the unit, his vital signs are stable and he has no complaints of pain. The nurse reviews the physician's orders. In addition to the medications already given, which medication does the nurse expect the physician to order?

Carvedilol (Coreg) Explanation: A client with suspected myocardial infarction should receive aspirin, nitroglycerin, morphine, and a beta-adrenergic blocker such as carvedilol. Digoxin treats arrhythmias; there is no indication that the client is having arrhythmias. Furosemide is used to treat signs of heart failure, which isn't indicated at this point. Nitroprusside increases blood pressure. This client has stable vital signs and isn't hypotensive.

A nurse is caring for a patient in the cardiovascular intensive care unit following a coronary artery bypass graft. Which of the following clinical findings requires immediate intervention by the nurse?

Central venous pressure reading of 1 mmHg Explanation: The central venous pressure (CVP) reading of 1 is low (2-6 mmHg) and indicates reduced right ventricular preload, commonly caused by hypovolemia. Hypovolemia is the most common cause of decreased cardiac output after cardiac surgery. Replacement fluids such as colloids, packed red blood cells, or crystalloid solutions may be prescribed. The other findings require follow-up by the nurse; however, addressing the CVP reading is the nurse's priority.

Which of the following nursing interventions should a nurse perform when a patient with cardiomyopathy receives a diuretic?

Check for dependent edema regularly Explanation: The nurse should monitor for dependent edema regularly if the patient with cardiomyopathy receives a diuretic. Oxygen is administered either continuously or when dyspnea or dysrhythmias develop. Bed rest is not necessary. The nurse should ensure that the patient's activity level is reduced and should sequence any activity that is slightly exertional between periods of rest.

A middle-aged male presents to the ED complaining of severe chest discomfort. Which of the following patient findings is most indicative of a possible MI?

Chest discomfort not relieved by rest or nitroglycerin Chest pain or discomfort not relieved by rest or nitroglycerin is associated with an acute MI. The other findings, although associated with ACS (acute coronary syndrome) or MI, may also occur with angina and, alone, are not indicative of an MI.

The nurse is aware that a client who has been diagnosed with Prinzmetal's angina will present with which of the following symptoms?

Chest pain that occurs at rest and usually in the middle of the night Explanation: A client with Prinzmetal's angina will complain of chest pain that occurs at rest, usually between 12 and 8 <SC>AM, is sporadic over 3-6 months, and diminishes over time. Client with stable angina generally experience chest pain that lasts 15 minutes or less and may radiate. Clients with Cardiac Syndrome X experience prolonged chest pain that accompanies exercise and is not always relieved by medication. Client with unstable angina experience chest pain of increased frequency, severity, and duration that is poorly relieved by rest or oral nitrates.

Which of the following medications is given to patients diagnosed with angina and is allergic to aspirin?

Clopidogrel (Plavix) Explanation: Plavix or Ticlid is given to patients who are allergic to aspirin or given in addition to aspirin to patients at high risk for MI. Norvasc, Cardizem, and Plendil are calcium channel blockers

Which of the following is a term used to describe the splitting or separating of fused cardiac valve leaflets?

Commissurotomy Explanation: Commissurotomy is the splitting or separating of fused cardiac valve leaflets. Annuloplasty is a repair of a cardiac valve's outer ring. Chordoplasty is repair of the stringy, tendinous fibers that connect the free edges of the atrioventricular valve leaflets to the papillary muscle. Valvuloplasty is a repair of a stenosed or regurgitant cardiac valve by commissurotomy, annuloplasty, leaflet repair, or chordoplasty.

An asymptomatic patient questions the nurse about mitral regurgitation and inquires about continuing exercises. Which of the following is the most appropriate nursing response?

Continue exercising until mild symptoms develop Explanation: Exercise is not limited until mild symptoms develop. It is not important for an asymptomatic patient to avoid exercise and to take ample rest after exercise.

A client comes to the emergency department (ED) complaining of precordial chest pain. In describing the pain, the client describes it as pressure with a sudden onset. What disease process would you suspect in this client?

Coronary artery disease The classic symptom of CAD is chest pain (angina) or discomfort during activity or stress. Such pain or discomfort typically is manifested as sudden pain or pressure that may be centered over the heart (precordial) or under the sternum (substernal). Raynaud's disease in the hands presents with symptoms of hands that are cold, blanched, and wet with perspiration. Cardiogenic shock is a complication of an MI. Venous occlusive disease occurs in the veins, not the arteries.

An electrocardiogram (ECG) taken during a routine checkup reveals that a client has had a silent myocardial infarction. Changes in which leads of a 12-lead ECG indicate damage to the left ventricular septal region?

Correct response: Leads V3 and V4 Explanation: Leads V3 and V4 record electrical events in the septal region of the left ventricle. Leads I, aVL, V5, and V6 record electrical events on the lateral surface of the left ventricle. Leads II, III, and aVF record electrical events on the inferior surface of the left ventricle. Leads V1 and V2 record electrical events on the anterior surface of the right ventricle and the anterior surface of the left ventricle.

The nurse, caring for a patient after cardiac surgery, is aware that fluid and electrolyte imbalance is a concern. Select the most immediate result that needs to be reported:

Correct response: Potassium level of 6 mEq/L Explanation: Changes in serum electrolytes should be immediately reported, especially a potassium level of 6 mEq/L. An elevated blood sugar is common postoperatively, and the weight gain isn't significant. The abnormal breath sounds are of concern, but the electrolyte imbalance is the most immediate condition that needs to be addressed.

During his annual physical exam, a 62-year-old male client reports experiencing chest pain and palpitations during and after his morning jogs. Family history reveals coronary artery disease. The nurse should instruct the client in the following to reduce the client's cardiac risk?

Correct response: Smoking cessation Explanation: The first line of defense for clients with CAD is lifestyle changes including smoking cessation, weight loss, stress management, and exercise. Clients with CAD should eat a balanced diet. Clients with CAD should exercise, as tolerated, to maintain a healthy weight. Antioxidant supplements, such as those containing vitamin E, beta carotene, and selenium, are not recommended because clinical trials have failed to confirm beneficial effects from their use.

The analgesic of choice for a hospitalized patient with an MI is morphine sulfate. An important nursing responsibility, prior to administering morphine, is to do which of the following?

Count the respiratory rate for bradypnea. The nurse should always check the respiratory rate prior to administering morphine sulfate. The drug should be withheld, and the health care provider notified, if the respiratory rate is below 16 breaths/minute.

The nurse is reviewing the laboratory results for a patient having a suspected myocardial infarction (MI). What cardiac-specific isoenzyme does the nurse observe for myocardial cell damage?

Creatine kinase MB Rationale: There are three creatine kinase (CK) isoenzymes: CK-MM (skeletal muscle), CK-MB (heart muscle), and CK-BB (brain tissue). CK-MB is the cardiac-specific isoenzyme; it is found mainly in cardiac cells and therefore increases when there has been damage to these cells. Elevated CK-MB is an indicator of acute MI; the level begins to increase within a few hours and peaks within 24 hours of an infarct.

A client seeks medical attention for dyspnea, chest pain, syncope, fatigue, and palpitations. A thorough physical examination reveals an apical systolic thrill and heave, along with a fourth heart sound (S4) and a systolic murmur. Diagnostic tests reveal that the client has hypertrophic cardiomyopathy (HCM). Which nursing diagnosis may be appropriate?

Decreased CO

During assessment of a client admitted for cardiomyopathy, the nurse notes the following symptoms: dyspnea on exertion, fatigue, fluid retention, and nausea. The initial appropriate nursing diagnosis is which of the following?

Decreased cardiac output A primary nursing diagnosis for cardiomyopathy is decreased cardiac output related to structural disorders caused by cardiomyopathy or to dysrhythmia from the disease process and medical treatments. Dyspnea on exertion, fatigue, and fluid retention are related to poor cardiac output. Nausea is related to poor perfusion of the gastrointestinal system. Autonomic dysreflexia is related to a spinal cord injury. Ineffective airway clearance relates to the inability to clear secretions from the airway, which is not an initial problem with cardiomyopathy. Disturbed sensory perception is related to specific senses and not to initial cardiomyopathy.

A client is recovering from coronary artery bypass graft (CABG) surgery. Which nursing diagnosis takes highest priority at this time

Decreased cardiac output related to depressed myocardial function, fluid volume deficit, or impaired electrical conduction For a client recovering from CABG surgery, Decreased cardiac output is the most important nursing diagnosis; anesthetics or a long cardiopulmonary bypass time may depress myocardial function, leading to decreased cardiac output. Other possible causes of decreased cardiac output in this client include fluid volume deficit and impaired electrical conduction. Anxiety, Disabled family coping, and Hypothermia may be relevant but take lower priority at this time; maintaining cardiac output is essential to sustaining the client's life.

A client is recovering from coronary artery bypass graft (CABG) surgery. Which nursing diagnosis takes highest priority at this time?

Decreased cardiac output related to depressed myocardial function, fluid volume deficit, or impaired electrical conduction Explanation: For a client recovering from CABG surgery, Decreased cardiac output is the most important nursing diagnosis; anesthetics or a long cardiopulmonary bypass time may depress myocardial function, leading to decreased cardiac output. Other possible causes of decreased cardiac output in this client include fluid volume deficit and impaired electrical conduction. Anxiety, Disabled family coping, and Hypothermia may be relevant but take lower priority at this time; maintaining cardiac output is essential to sustaining the client's life.

A nurse is caring for a patient who experienced an MI. The patient is ordered metoprolol (Lopressor). The nurse understands that the therapeutic effect of this medication is which of the following?

Decreases resting heart rate The therapeutic effects of beta-adrenergic blocking agents such as metoprolol are to reduce the myocardial oxygen consumption by blocking beta-adrenergic sympathetic stimulation to the heart. The result is reduced heart rate, slowed conduction of impulses through the conduction system, decreased blood pressure, and reduced myocardial contractility to balance the myocardial oxygen needs and amount of oxygen available.

The nurse is administering a calcium channel blocker to a patient who has symptomatic sinus tachycardia at a rate of 132 bpm. What is the anticipated action of the drug for this patient?

Decreases the sinoatrial node automaticity Calcium channel blockers have a variety of effects on the ischemic myocardium. These agents decrease sinoatrial node automaticity and atrioventricular node conduction, resulting in a slower heart rate and a decrease in the strength of myocardial contraction (negative inotropic effect).

A client is recovering from coronary artery bypass graft (CABG) surgery. The nurse knows that for several weeks after this procedure, the client is at risk for certain conditions. During discharge preparation, the nurse should advise the client and his family to expect which common symptom that typically resolves spontaneously?

Depression Explanation: For the first few weeks after CABG surgery, clients commonly experience depression, fatigue, incisional chest discomfort, dyspnea, and anorexia. Depression typically resolves without medical intervention. However, the nurse should advise family members that symptoms of depression don't always resolve on their own. They should make sure they recognize worsening symptoms of depression and know when to seek care. Ankle edema seldom follows CABG surgery and may indicate right-sided heart failure. Because this condition is a sign of cardiac dysfunction, the client should report ankle edema at once. Memory lapses reflect neurologic rather than cardiac dysfunction. Dizziness may result from decreased cardiac output, an abnormal condition following CABG surgery. This symptom warrants immediate physician notification.

Patient with myocarditis are sensitive to which of the following medications?

Digitalis Explanation: The nurse must closely monitor these patients for digitalis toxicity, which is evidenced by dysrhythmia, anorexia, nausea, vomiting, headache, and malaise. If the cause of the myocarditis is hemolytic streptococci, penicillin will be given. The use of corticosteroids remains controversial.

A patient is exhibiting digitalis toxicity. Which of the following medications would the nurse expect to be ordered for this patient?

Digoxin

Which of the following medications reverses digitalis toxicity?

Digoxin immune FAB (Digibind)

Which drug is most commonly used to treat cardiogenic shock?

Dopamine (Intropin)

You are the triage nurse in a walk-in clinic when a diabetic client visits the clinic and asks you to take her blood pressure (BP). The measurements are 150/90 mm Hg. Which of the following would the nurse expect as the treatment to normalize the client's BP?

Drug therapy

Frequently, what is the earliest symptom of left-sided heart failure?

Dyspnea on exertion

The diagnosis of aortic regurgitation (AR) is confirmed by which of the following?

Echocardiography Explanation: Diagnosis is confirmed by echocardiography. Cardiac catheterization is not necessary in most patients with AR. Exercise stress testing will assess functional capacity and symptom response. A myocardial biopsy may be performed to analyze myocardial tissue cells in patients with cardiomyopathy.

A patient has been prescribed a digitalis preparation for heart failure. Which of the following should you, as her nurse, closely monitor when caring for this client?

Electrolyte and water loss

A client comes to the emergency department complaining of chest pain. An electrocardiogram (ECG) reveals myocardial ischemia and an anterior-wall myocardial infarction (MI). Which ECG characteristic does the nurse expect to see?

Elevated ST segment Explanation: Ischemic myocardial tissue changes cause elevation of the ST segment, an inverted T wave, and a pathological Q wave. A prolonged PR interval occurs with first-degree heart block, the least dangerous atrioventricular heart block; this disorder may arise in healthy people but sometimes results from drug toxicity, electrolyte or metabolic disturbances, rheumatic fever, or chronic degenerative disease of the conduction system. An absent Q wave is normal; an MI may cause a significant Q wave. A widened QRS complex indicates a conduction delay in the His-Purkinje system.

A female client returns for a follow-up visit to the cardiologist 4 days after a trip to the ED for sudden shortness of breath and abdominal pain. The nurse realizes the client had a myocardial infarction because the results from the blood work drawn in the hospital shows:

Elevated troponin levels Troponin is present only in myocardial tissue; therefore, it is the gold standard for determining heart damage in the early stages of an MI. LDH1 and LDH2 may be elevated in response to cardiac or other organ damage during an MI. Myoglobin is a biomarker that rises in 2 to 3 hours after heart damage during an MI. C-reactive protein, erythrocyte sedimentation rate, and the WBC count increase on about the third day following MI because of the inflammatory response that the injured myocardial cells triggered. These levels would not be elevated during the MI event.

The nursing priority of care for a client exhibiting signs and symptoms of coronary artery disease should be to:

Enhance myocardial oxygenation

A nurse is caring for a client with aortic stenosis whose compensatory mechanisms of the heart have begun to fail. The nurse will monitor the client carefully for which initial symptoms?

Exertional dyspnea, orthopnea, pulmonary edema Explanation: When symptoms develop, clients with aortic stenosis usually first have exertional dyspnea, caused by increased pulmonary venous pressure from left heart failure. Orthopnea, paroxysmal nocturnal dyspnea, dizziness, and pulmonary edema may also occur. Nausea and vomiting may be signs of gastrointestinal congestion, but would be related to right heart failure, which does not occur initially with aortic stenosis.

The nurse is preparing to administer warfarin (Coumadin), an oral anticoagulant, to a patient with a mechanical valve replacement. The patient's INR is 2.7. Which action should the nurse take?

Explanation: Patients with mechanical valve replacements who take warfarin (Coumadin) usually have individualized target international normalized ratios (INRs) between 2.0 and 3.5. The nurse would give the medication as ordered.

A patient presents to the emergency room with characteristics of atherosclerosis. What characteristics would the patient display?

Fatty deposits in the lumen of arteries Atherosclerosis is a condition in which the lumen of arteries fill with fatty deposits called plaque. Therefore, options B, C, and D are incorrect.

Which signs and symptoms accompany a diagnosis of pericarditis?

Fever, chest discomfort, and elevated erythrocyte sedimentation rate (ESR). Explanation: The classic signs and symptoms of pericarditis include fever, positional chest discomfort, nonspecific ST-segment elevation, elevated ESR, and pericardial friction rub. Low urine output secondary to left ventricular dysfunction lethargy, anorexia, heart failure and pitting edema, result from acute renal failure.

What are the side effects of nitroglycerin?

Flushing, throbbing headache, and tachycardia.

Which of the following is a nonmodifiable risk factor for coronary artery disease (CAD)?

Gender

A physician treating a client in the cardiac care unit for atrial arrhythmia orders metoprolol (Lopressor), 25 mg P.O. two times per day. Metoprolol inhibits the action of sympathomimetics at beta1-receptor sites. Where are these sites mainly located?

Heart Most beta1-receptor sites are located in the heart. Beta2-receptor sites are located in the uterus, blood vessels, and bronchi.

A nurse is caring for a client experiencing dyspnea, dependent edema, hepatomegaly, crackles, and jugular vein distention. What condition should the nurse suspect?

Heart failure

Which of the following therapies are for patient who have advanced heart failure (HF) after all other therapies have failed?

Heart transplant

A nursing student is caring for a client with end-stage cardiomyopathy. The client's spouse asks the student to clarify one of the last treatment options available that the physician mentioned. After checking with the primary nurse, the student would most likely discuss which of the following?

Heart transplantation Explanation: When heart failure progresses and medical treatment is no longer effective, surgical intervention, including heart transplantation, is considered. The other three choices have to do with failing valves and valve repairs.

A nurse is monitoring the vital signs and blood results of a 53-year-old male patient who is receiving anti-coagulation therapy. Which of the following does the nurse identify as a major indication of concern?

Hematocrit of 30% Hematocrit is a measurement of the proportion of blood volume that is occupied by red blood cells. A lowered hematocrit can imply internal bleeding.

The lab values of a patient diagnosed with coronary artery disease (CAD) have just come back from the lab. His low-density lipoprotein (LDL) level is 112 mg/dL. This lab value is what?

High LDL

A 62-year-old male client who works as a stockbroker reports chest pain and palpitations during and after his morning jogs. The client's family history includes CAD. His lipid profile reveals his LDL level to be 122 mg/dL. The nurse is aware that this client has which of the following?

High LDL level

Often, women and elderly do not have the typical chest pain associated with a myocardial infarction. Some report vague symptoms (fatigue, abdominal pain), which can lead to misdiagnosis. Some older adults may experience little or no chest pain. Gender is not a contributing factor for fatal occurrence but rather a result of symptoms association.

High-density lipoprotein (HDL), 80 mg/dL A fasting lipid profile should demonstrate the following values (Alberti et al., 2009): LDL cholesterol less than 100 mg/dL (less than 70 mg/dL for very high-risk patients); total cholesterol less than 200 mg/dL; HDL cholesterol greater than 40 mg/dL for males and greater than 50 mg/dL for females; and triglycerides less than 150 mg/dL.

The nurse is discussing risk factors for developing CAD with a patient in the clinic. Which results would indicate that the patient is not at significant risk for the development of CAD?

High-density lipoprotein (HDL), 80 mg/dL Rationale: A fasting lipid profile should demonstrate the following values (Alberti et al., 2009): LDL cholesterol less than 100 mg/dL (less than 70 mg/dL for very high-risk patients); total cholesterol less than 200 mg/dL; HDL cholesterol greater than 40 mg/dL for males and greater than 50 mg/dL for females; and triglycerides less than 150 mg/dL.

A white male, age 43, with a tentative diagnosis of infective endocarditis is admitted to an acute care facility. His medical history reveals diabetes mellitus, hypertension, and pernicious anemia; he underwent an appendectomy 20 years earlier and an aortic valve replacement 2 years before this admission. Which history finding is a major risk factor for infective endocarditis?

History of aortic valve replacement Explanation: A heart valve prosthesis such as an aortic valve replacement is a major risk factor for infective endocarditis. Other risk factors include a history of heart disease (especially mitral valve prolapse), chronic debilitating disease, I.V. drug abuse, and immunosuppression. Although race, age, and a history of diabetes mellitus may predispose a person to cardiovascular disease, they aren't major risk factors for infective endocarditis.

When the postcardiac surgery patient demonstrates restlessness, nausea, weakness, and peaked T waves, the nurse reviews the patient's serum electrolytes anticipating which abnormality?

Hyperkalemia Hyperkalemia is indicated by mental confusion, restlessness, nausea, weakness, and dysrhythmias (tall, peaked T waves). Hypercalcemia would likely be demonstrated by asystole. Hypomagnesemia would likely be demonstrated by hypotension, lethargy, and vasodilation. Hyponatremia would likely be indicated by weakness, fatigue, and confusion without change in T-wave formation

A nurse is caring for a patient post cardiac surgery. Upon assessment, the patient appears restless and is complaining of nausea and weakness. The patient's ECG reveals peaked T waves. The nurse reviews the patient's serum electrolytes anticipating which of the following abnormalities?

Hyperkalemia Hyperkalemia is indicated by mental confusion, restlessness, nausea, weakness, and dysrhythmias (tall, peaked T waves). Hypercalcemia would likely be demonstrated by asystole. Hypomagnesemia would likely be demonstrated by hypotension, lethargy, and vasodilation. Hyponatremia would likely be indicated by weakness, fatigue, and confusion without change in T-wave formation.

When the postcardiac surgical patient demonstrates vasodilation, hypotension, hyporeflexia, slow gastrointestinal motility (hypoactive bowel sounds), lethargy, and respiratory depression, the nurse suspects which of the following electrolyte imbalances?

Hypermagnesemia Explanation: Untreated hypomagnesemia may result in coma, apnea, and cardiac arrest. Signs and symptoms of hypokalemia include signs of digitalis toxicity and dysrhythmias (U wave, AV block, flat or inverted T waves). Signs of hyperkalemia include: mental confusion, restlessness, nausea, weakness, paresthesias of extremities, dysrhythmias (tall, peaked T waves; increased amplitude, widening QRS complex; prolonged QT interval). Signs and symptoms of hypomagnesemia include: paresthesias, carpopedal spasm, muscle cramps, tetany, irritability, tremors, hyperexcitability, hyperreflexia, cardiac dysrhythmias (prolonged PR and QT intervals, broad flat T waves), disorientation, depression, and hypotension

In which type of cardiomyopathy does the heart muscle actually increase in size and mass weight, especially along the septum?

Hypertrophic Explanation: Because of the structural changes, hypertrophic cardiomyopathy had also been called idiopathic hypertrophic subaortic stenosis (IHSS) or asymmetric septal hypertrophy (ASH). Restrictive cardiomyopathy is characterized by diastolic dysfunction caused by rigid ventricular walls that impair ventricular stretch and diastolic filling. Arrhythmogenic right ventricular cardiomyopathy (ARVC) occurs when the myocardium of the right ventricle is progressively infiltrated and replaced by fibrous scar and adipose tissue.

Which of the following is inconsistent as a condition related to metabolic syndrome?

Hypotension

A new surgical patient who has undergone a coronary artery bypass graft (CABG) is receiving opioids for pain control. The nurse must be alert to adverse effects of opioids. Which of the following effects would be important for the nurse to document?

Hypotension The patient is observed for any adverse effects of opioids, which may include respiratory depression, hypotension, ileus, or urinary retention. If serious side effects occur, an opioid antagonist, such as Narcan, may be used.

Which of the following in an inconsistent manifestation of metabolic syndrome?

Hypotension Explanation: Metabolic syndrome consists of insulin resistance, dyslipidemia, hypertension, and chronic inflammation.

Which New York Heart Association classification of heart failure has a poor prognosis and includes symptoms of cardiac insufficiency at rest?

IV

A client with venous insufficiency is instructed to exercise, apply elastic stockings, and elevate the extremities. Which is the primary benefit for this nursing management regime?

Improve venous return The major goal in management of venous insufficiency is to promote venous circulation. Arterial flow improvement is not the goal of treatment for this disorder. Venous valves that are incompetent cannot be strengthened. Venous congestion is a complication of venous insufficiency.

Which of the following would be inconsistent as criterion of extubation in the patient who has undergone a coronary artery bypass graft (CABG)?

Inability to speak. Explanation: Before being extubated, the patient should have cough and gag reflexes and stable vital signs; be able to life the head off the bed or give firm hand grasps; have adequate vital capacity, negative inspiratory force, and minute volume appropriate for body size; and have acceptable ABG levels while breathing without the assistance of the ventilator. Inability to talk is expected when intubated with an endotracheal tube.

When the nurse notes that the post cardiac surgery patient demonstrates low urine output (< 25 mL/hr) with high specific gravity (> 1.025), the nurse suspects:

Inadequate fluid volume Rationale: Urine output of less than 25 mL/hr may indicate a decrease in cardiac output. A high specific gravity indicates increased concentration of solutes in the urine, which occurs with inadequate fluid volume. Indices of normal glomerular filtration are output of 25 mL or greater per hour and specific gravity between 1.010 and 1.025. Overhydration is manifested by high urine output with low specific gravity. The anuric patient does not produce urine.

Which of the following is the most important postoperative assessment parameter for patients undergoing cardiac surgery?

Inadequate tissue perfusion Explanation: The nurse must assess the patient for signs and symptoms of inadequate tissue perfusion, such as a weak or absent pulse, cold or cyanotic extremities, or mottling of the skin. Although the nurse does assess blood sugar and mental status, tissue perfusion is the higher priority. Assessing for activity intolerance, while important later in the recovery period, is not essential in the immediate postoperative period for patients undergoing cardiac surgery.

A patient's elevated cholesterol levels are being managed with Lipitor, 40 mg daily. The nurse practitioner reviews the patient's blood work every 6 months before renewing the prescription. The nurse explains to the patient's daughter that this is necessary because of a major side effect of Lipitor that she is checking for. What is that side-effect?

Increased liver enzymes Myopathy and increased liver enzymes are significant side effects of the statins, HMG-CoA reductase inhibitors that are used to affect lipoprotein metabolism.

A patient has been recently placed on nitroglycerin. Which of the following should be included in the patient teaching plan?

Instruct the patient on side effects of flushing, throbbing headache, and tachycardia. Explanation: The patient should be instructed about side effects of the medication, which include flushing, throbbing headache, and tachycardia. The patient should renew the nitroglycerin supply every 6 months. If the pain is severe, the patient can crush the tablet between the teeth to hasten sublingual absorption. Tablets should never be removed and stored in metal or plastic pillboxes. Nitroglycerin is very unstable and should be carried in its original container.

What can a low hematocrit level indicate?

Internal bleeding

The nurse is educating a patient diagnosed with angina pectoris about the difference between the pain of angina and a myocardial infarction (MI). How should the nurse describe the pain experienced during an MI? (Select all that apply.)

It is substernal in location. It is viselike and radiates to the shoulders and arms. It is sudden in onset and prolonged in duration Chest pain that occurs suddenly, continues despite rest and medication, is substernal, and is sometimes viselike and radiating to the shoulders and arms is associated with an MI. Angina pectoris pain is generally relieved by rest and nitroglycerin.

Which of the following is a characteristic of right-sided heart failure?

JVD (Jugular vein distention) Explanation: JVD is a characteristic of right-sided heart failure. Dyspnea, pulmonary crackles, and cough are manifestations of left-sided heart failure.

A new client has been admitted with right-sided heart failure. The nurse knows to look for which of the following assessment findings when assessing this client?

Jugular vein distention

A 44-year-old client has a history of hypertension. As her nurse, you engage her in client education to make her aware of structures that regulate arterial pressure. Which of the following structures are a component of that process?

Kidneys

An 83-year-old client is undergoing lipid profile studies in an effort to determine a proper nutritional balance for his CAD. In his lipid profile, his LDL is greater than his HDL. Why is this a risk factor for this client?

LDL sticks to the arteries

Which of the following medications is categorized as a loop diuretic?

Lasix

A client who suffered blunt chest trauma in a motor vehicle accident complains of chest pain, which is exacerbated by deep inspiration. On auscultation, the nurse detects a pericardial friction rub — a classic sign of acute pericarditis. The physician confirms acute pericarditis and begins appropriate medical intervention. To relieve chest pain associated with pericarditis, which position should the nurse encourage the client to assume?

Leaning forward while sitting Explanation: The nurse should encourage the client to lean forward, because this position causes the heart to pull away from the diaphragmatic pleurae of the lungs, helping relieve chest pain caused by pericarditis. The semi-Fowler's, supine, and prone positions don't cause this pulling-away action and therefore don't relieve chest pain associated with pericarditis.

Incomplete closure of the mitral valve results in backflow of blood from the:

Left ventricle to left atrium Explanation: Incompetent closure of the mitral valve can result from disease processes that alter valve leaflets, mitral annulus, chordae tendineae, and the papillary muscle. When mitral valve leaflets thicken, fibrose, and contract, they cannot close completely during systole. This forces blood backward from the left ventricle into the left atrium during systole.

A nursing student is caring for a client with end-stage cardiomyopathy. The client's spouse asks the nurse to clarify one of the last treatment options available that the physician mentioned earlier. After checking with the primary nurse, the nursing student would most likely discuss which of the following?

Left ventricular assist device Explanation: When heart failure progresses and medical treatment is no longer effective, surgical intervention, including heart transplantation, is considered. Because of the limited number of organ donors, many clients die waiting. In some cases, a left ventricular assist device is implanted to support the failing heart until a suitable donor becomes available. The other three choices have to do with failing valves and valve repairs.

For patients diagnosed with aortic stenosis, digoxin would be ordered for which of the following clinical manifestations?

Left ventricular dysfunction Explanation: Digoxin may be used to treat left ventricular dysfunction, and diuretics may be used for dyspnea. Nitrates may be prescribed for the treatment of angina, but must be used with caution due to the risk of orthostatic hypotension and syncope.

Cholesterol should make up ______mg/day

Less than 200

A nurse is teaching a client who receives nitrates for the relief of chest pain. Which instruction should the nurse emphasize?

Lie down or sit in a chair for 5 to 10 minutes after taking the drug. Nitrates act primarily to relax coronary smooth muscle and produce vasodilation. They can cause hypotension, which makes the client dizzy and weak. The nurse should instruct the client to lie down or sit in a chair for 5 to 10 minutes after taking the drug. Nitrates are taken at the first sign of chest pain and before activities that might induce chest pain. Sublingual nitroglycerin is taken every 5 minutes for three doses. If the pain persists, the client should seek medical assistance immediately. Nitrates must be stored in a dark place in a closed container because sunlight causes the medication to lose its effectiveness. Alcohol is prohibited because nitrates may enhance the effects of the alcohol.

The nurse is caring for an 82-year-old male client who has come to the clinic for a yearly physical. When assessing the client, the nurse notes the blood pressure (BP) is 140/93. The nurse knows that in older clients what happens that may elevate the systolic BP?

Loss of arterial elasticity

A patient is receiving anticoagulant therapy. The nurse should be alert to potential signs and symptoms of external or internal bleeding, as evidenced by which of the following?

Low blood pressure The patient receiving anticoagulation therapy should be monitored for signs and symptoms of external and internal bleeding, such as low blood pressure, increased heart rate, and decreased serum hemoglobin and hematocrit.

As part of health education for a patient with an abnormal fasting lipid profile, the nurse explains that an excess of this lipid leads to the formation of plaque in the arteries. Identify the lipid.

Low-density lipoproteins (LDL) Rationale: When there is an excess of LDL, these particles adhere to vulnerable points in the arterial endothelium. Here, macrophages ingest then, leading to the formation of foam cells and the beginning of plaque formation. A harmful effect is exerted on the coronary vasculature because the small LDL particles can be easily transported into the vessel lining.

Ralph Wilson, is a 52-year-old client in the hospital unit where you practice nursing. He is being treated for myocarditis. Which of the following nursing interventions should you perform to reduce cardiac workload in a client with myocarditis?

Maintain the client on bed rest Explanation: The nurse should maintain the client on bed rest to reduce cardiac workload and promote healing.

Which of the following nursing interventions should a nurse perform to reduce cardiac workload in a patient diagnosed with myocarditis?

Maintain the patient on bed rest Explanation: The nurse should maintain the patient on bed rest to reduce cardiac workload and promote healing. The nurse should administer supplemental oxygen to relieve tachycardia that may develop from hypoxemia. If the patient has a fever, the nurse should administer a prescribed antipyretic along with independent nursing measures like minimizing layers of bed linen, promoting air circulation and evaporation of perspiration, and offering oral fluids. The nurse should elevate the patient's head to promote maximal breathing potential.

A nurse completed a physical exam for an insurance company. The nurse noted a cluster of abnormalities that she knew was considered a major risk factor for coronary artery disease. Choose that condition.

Metabolic syndrome Metabolic syndrome includes three of six conditions that are recognized as a major risk factor for CAD. Insulin resistance is part of the syndrome but the patient may not yet have diabetes.

Which of the following techniques is used to surgically revascularize the myocardium?

Minimally invasive direct coronary bypass Explanation: There are several techniques used to surgically revascularize the myocardium; one of them is minimally invasive direct coronary bypass. Balloon bypass is not used to revascularize the myocardium. If the patient is experiencing acute pain in the leg, peripheral bypass is performed. Gastric bypass is a surgical procedure that alters the process of digestion.

A 73-year-old client has been admitted to the cardiac step-down unit where you practice nursing. After diagnostics, she was brought to your unit with acute pulmonary edema. Which of the following symptoms would you expect to find during your assessment?

Moist, gurgling respirations Explanation: Clients with acute pulmonary edema experience sudden dyspnea, wheezing, orthopnea, cough, cyanosis, and tachycardia. Respirations sound moist or gurgling.

Which of the following discharge instructions for self-care should the nurse provide to a patient who has undergone a percutaneous transluminal coronary angioplasty (PTCA) procedure?

Monitor the site for bleeding or hematoma. The nurse provides certain discharge instructions for self-care, such as monitoring the site for bleeding or development of a hard mass indicative of hematoma. A nurse does not advise the patient to clean the site with disinfectants or refrain from sexual activity for one month.

Upon discharge from the hospital, patients diagnosed with a myocardial infarction (MI) must be placed on all of the following medications except:

Morphine IV Upon patient discharge, there needs to be documentation that the patient was discharged on a statin, an ACE or angiotensin receptor blocking agent (ARB), and aspirin. Morphine IV is used for these patients to reduce pain and anxiety. The patient would not be discharged with IV morphine.

The nurse is caring for a patient who is having chest pain associated with a myocardial infarction (MI). What medication should the nurse administer intravenously to reduce pain and anxiety?

Morphine sulfate

Severe chest pain is reported by a client during an acute myocardial infarction. Which of the following is the most appropriate drug for the nurse to administer?

Morphine sulfate (Morphine) Explanation: Morphine not only decreases pain perception and anxiety but also helps to decrease heart rate, blood pressure, and demand for oxygen. Nitrates are administered for vasodilation and pain control in clients with angina-type pain, but oral forms (such as Isordil) have a large first-pass effect, and transdermal patch is used for long-term management. Demerol is a synthetic opioid usually reserved for treatment of postoperative or migraine pain.

Troponin is present only on..

Myocardial tissue

The nurse knows to review laboratory tests for cardiac biomarkers for a patient suspected of suffering an MI. The nurse knows that all can be detected within hours of an injury. Which of the following is the earliest marker?

Myoglobin Myoglobin is a heme protein that transports oxygen. Its levels can increase as early as 1 hour after an MI. Negative results are an excellent parameter for ruling out an acute MI. The other biomarker choices start to increase in 2 to 4 hours.

The nurse is caring for a male patient who is being evaluated for lipid-lowering medication. The patient's laboratory results reveal the following: Total cholesterol: 230 mg/dL, LDL: 120 mg/dL, and a triglyceride level of 310 mg/dL. Which of the following classes of medications would be most appropriate for the patient based on his laboratory findings?

Nicotinic acids The most appropriate class of medications based on the patient's laboratory findings would be nicotinic acids. This class of medications is prescribed for patients with: minimally elevated cholesterol and LDL levels or as an adjunct to a statin when the lipid goal has not been has not been achieved and triglyceride (TG) levels are elevated.

A patient presents to the ED complaining of anxiety and chest pain after shoveling heavy snow that morning. The patient says that he has not taken nitroglycerin for months but did take three nitroglycerin tablets and although the pain is less, "They did not work all that well. " The patient shows the nurse the nitroglycerin bottle and the prescription was filled 12 months ago. The nurse anticipates which of the following physician orders?

Nitroglycerin SL Nitroglycerin is volatile and is inactivated by heat, moisture, air, light, and time. Nitroglycerin should be renewed every 6 months to ensure full potency. The client's tablets were expired and the nurse should anticipate administering nitroglycerin to assess if the chest pain subsides. The other choices may be ordered at a later time, but the priority is to relieve the patient's chest pain.

A physician admits a client with a history of I.V. drug abuse to the medical-surgical unit for evaluation for infective endocarditis. Nursing assessment is most likely to reveal that this client has:

Osler's nodes and splinter hemorrhages. Explanation: Infective endocarditis occurs when an infectious agent enters the bloodstream, such as from I.V. drug abuse or during an invasive procedure or dental work. Typical assessment findings in clients with this disease include Osler's nodes (red, painful nodules on the fingers and toes), splinter hemorrhages, fever, diaphoresis, joint pain, weakness, abdominal pain, a new or altered heart murmur, and Janeway's lesions (small, hemorrhagic areas on the fingers, toes, ears, and nose). Retrosternal pain that worsens when the client is supine, pulsus paradoxus, and pericardial friction rub are common findings in clients with pericarditis, not infective endocarditis.

What is variant angina described as?

Pain at rest with reversible ST-segment elevation and is thought to be caused by coronary artery vasospasm

A client with Raynaud's disease complains of cold and numbness in the fingers. Which of the following would the nurse identify as an early sign of vasoconstriction?

Pallor Pallor is the initial symptom in Raynaud's followed by cyanosis and aching pain. Gangrene can occur with persistent attacks and interference of blood flow. Clubbing of the fingers is a symptom associated with chronic oxygen deprivation to the distal phalanges.

Postpericardiotomy syndrome may occur in patients who undergo cardiac surgery. The nurse should be alert to which of the following clinical manifestations associated with this syndrome?

Pericardial friction rub The syndrome is characterized by fever, pericardial pain, pleural pain, dyspnea, pericardial effusion, pericardial friction rub, and arthralgia. Leukocytosis (elevated WBCs) occurs, along with elevation of the ESR.

A patient in the recovery room after cardiac surgery begins to have extremity paresthesia, peaked T waves, and mental confusion. What type of electrolyte imbalance does the nurse suspect this patient is having?

Potassium Hyperkalemia (high potassium) can result in the following ECG changes: tall peaked T waves, wide QRS, and bradycardia. The nurse should be prepared to administer a diuretic or an ion-exchange resin (sodium polystyrene sulfonate [Kayexalate]); IV sodium bicarbonate, or IV insulin and glucose. Imbalances in the other electrolytes listed would not result in peaked T waves.

A physician orders digoxin (Lanoxin) for a client with heart failure. During digoxin therapy, which laboratory value may predispose the client to digoxin toxicity?

Potassium level of 2.8 mEq/L

The nurse, caring for a patient after cardiac surgery, is aware that fluid and electrolyte imbalance is a concern. Select the most immediate result that needs to be reported.

Potassium level of 6 mEq/L

A patient diagnosed with coronary artery disease is being placed on nitroglycerin. The nurse understands that the premise behind administration of nitrates in this patient population includes which of the following?

Preload is reduced. Explanation: Nitroglycerin dilates primarily the veins, and in higher dosages, also the arteries. Dilation of the veins causes venous pooling of the blood throughout the body. As a result, less blood returns to the heart, and filling pressure (preload ) is reduced. Nitroglycerine is administered to reduce myocardial oxygen consumption, which decreases ischemia and relieves pain

A patient with pericarditis is experiencing cardiac tamponade. Which collaborative intervention should the nurse anticipate for this patient?

Prepare for pericardiocentesis. Explanation: The nurse notifies the physician immediately and prepares to assist with pericardiocentesis, the emergency treatment for cardiac tamponade. Cardiac enzymes may be elevated but would not be ordered nor would a 12-lead ECG. The nurse's assessment of the lungs and heart is not a collaborative, but an independent action.

Which of the following mitral valve conditions generally produces no symptoms?

Prolapse Explanation: Mitral valve prolapse is a deformity that usually produces no symptoms and has been diagnosed more frequently in recent years, probably as a result of improved diagnostic methods. Mitral valve stenosis usually causes progressive fatigue. Mitral valve regurgitation, in its acute stage, usually presents as severe heart failure. Mitral valve infection, when acute, will produce symptoms typical of infective endocarditis.

Which of the following medications is an antidote to heparin?

Protamine sulfate Rationale: Protamine sulfate is known as the antagonist to heparin. Alteplase is a thrombolytic agent. Clopidogrel (Plavix) is an antiplatelet medication that is given to reduce the risk of thrombus formation post coronary stent placement. The antiplatelet effect of aspirin does not reverse the effects of heparin.

A client has had oral anticoagulation ordered. What should you monitor for when your client is taking oral anticoagulation?

Prothrombin time (PT) or international normalized ratio (INR) The nurse should monitor PT or INR when oral anticoagulation is prescribed. Vascular sites for bleeding, urine output, and hourly IV infusions are generally monitored in all clients.

A client is admitted to the hospital with systolic left-sided heart failure. The nurse knows to look for which of the following assessment findings for this client?

Pulmonary congestion

A physician has scheduled a client with mitral stenosis for mitral valve replacement. Which condition may arise as a complication of mitral stenosis?

Pulmonary hypertension Explanation: Mitral stenosis, or severe narrowing of the mitral valve, impedes blood flow through the stenotic valve, increasing pressure in the left atrium and pulmonary circulation. These problems may lead to low cardiac output, pulmonary hypertension, edema, and right-sided (not left-sided) heart failure. Other potential complications of mitral stenosis include mural thrombi, pulmonary hemorrhage, and embolism to vital organs. Myocardial ischemia may occur in a client with coronary artery disease. Left ventricular hypertrophy is a potential complication of aortic stenosis.

Which valve lies between the right ventricle and the pulmonary artery?

Pulmonic Explanation: The pulmonic valve is a semilunar valve located between the right ventricle and the pulmonary artery. The tricuspid valve is an atrioventricular valve located between the right atrium and right ventricle. The mitral valve is an atrioventricular valve located between the left atrium and left ventricle. Chordae tendineae anchor the valve leaflets to the papillary muscle and ventricular wall.

Which of the following would be inconsistent as a lifestyle change directive for the patient diagnosed with heart failure?

Push fluids

A client is being evaluated for coronary artery disease (CAD) and is scheduled for an electron beam computed tomography. The nurse understands that the primary advantage of this radiologic test is which of the following?

Quantifies calcified plaque The primary advantage of EBCT is to detect and quantify calcified plaque in the coronary arteries even before symptoms arise. EBCT is noninvasive and provides clearer images with less exposure to radiation than a CT scan but not the primary reason for use.

After undergoing cardiac surgery, a patient discovers a painless lump and complains to the nurse about the same. Which of the following is the most important nursing intervention for this patient?

Reassure the patient by informing him or her that the lump will disappear with time. Explanation: The nurse will reassure the patient by informing him or her that the lump will disappear with time and will not require surgery, drug therapy, or a visit to the physician.

A central venous pressure (CVP) reading of 1 indicates what?

Reduced right ventricular preload, commonly caused by hypovolemia.

Which of the following is an action of the intra-aortic balloon pump (IABP)?

Reduction of left ventricular afterload Explanation: The IABP decreases the workload of the heart by reducing left ventricular afterload. Additionally, it improves coronary artery blood flow by increasing coronary artery perfusion pressure. It does not reduce left or right ventricular preload.

The nurse is part of a triage team that is assessing a patient to determine if his chest pain is a manifestation of angina pectoris or an MI. The nurse knows that a primary distinction is that the pain of angina is:

Relieved by rest and nitroglycerin

Following the assessment of a patient with suspected pericarditis, the nursing student would determine which of the following findings to be most characteristic?

Reports of constant chest pain Explanation: The most characteristic symptom of pericarditis is chest pain. The pain or discomfort usually remains fairly constant, but it may worsen with deep inspiration and when lying down or turning. Other signs may include a mild fever, increased WBC count, anemia, and an elevated ESR or C-reactive protein level. Dyspnea and other signs and symptoms of heart failure may occur.

Which of the following types of cardiomyopathy are characterized by diastolic dysfunction caused by rigid ventricular walls that impair diastolic filling and ventricular stretch.

Restrictive cardiomyopathy (RCM) Explanation: RCM is characterized by diastolic dysfunction caused by rigid ventricular walls that impair diastolic filling and ventricular stretch. Hypertrophic cardiomyopathy occurs when the heart muscle asymmetrically increases in size and mass, especially along the septum. Dilated cardiomyopathy is distinguished by significant dilation of the ventricles without simultaneous hypertrophy. Arrhythmogenic right ventricular cardiomyopathy occurs when the myocardium of the right ventricle is progressively infiltrated and replaced by fibrous scar and adipose tissue.

What is the most common cause of mitral stenosis?

Rheumatic endocarditis Explanation: Mitral stenosis is most often caused by rheumatic endocarditis, which progressively thickens the mitral valve leaflets and chordate tendineae. In adults, aortic stenosis is often a result of degenerative calcifications. Congestive heart failure and myocardial infarction are not the most common cause of mitral stenosis.

A nurse is obtaining a history from a new client in the cardiovascular clinic. When investigating for childhood diseases and disorders associated with structural heart disease, which finding should the nurse consider significant?

Rheumatic fever Explanation: Childhood diseases and disorders associated with structural heart disease include rheumatic fever and severe streptococcal (not staphylococcal) infections. Croup — a severe upper airway inflammation and obstruction that typically strikes children ages 3 months to 3 years — may cause latent complications, such as ear infection and pneumonia. However, it doesn't affect heart structures. Likewise, medullary sponge kidney, characterized by dilation of the renal pyramids and formation of cavities, clefts, and cysts in the renal medulla, may eventually lead to hypertension but doesn't damage heart structures.

The nurse is providing discharge teaching for a client with rheumatic endocarditis but no valvular dysfunction. On which nursing diagnosis should the nurse focus her teaching?

Risk for infection Explanation: Clients with endocarditis have a Risk for infection. The nurse should stress to the client that he'll need to continue antibiotics for a minimum of 5 years and that he'll need to take prophylactic antibiotics before invasive procedures for life. There is no indication that the client has Chronic pain or Impaired memory. Because the client doesn't have valvular damage, Impaired gas exchange doesn't apply.

Which of the following symptoms occurs in the patient diagnosed with mitral regurgitation when pulmonary congestion occurs?

SOB Explanation: If pulmonary congestion occurs, the patient with mitral regurgitation develops shortness of breath. A loud, blowing murmur often is heard throughout ventricular systole at the heart's apex. Hypertension may develop when reduced cardiac output triggers the renin-angiotensin-aldosterone cycle. Tachycardia is a compensatory mechanism when stroke volume decreases.

A client, newly admitted to the nursing unit, has a primary diagnosis of renal failure. When assessing the client, the nurse notes a blood pressure (BP) of 180/100. The nurse knows that this is what kind of hypertension?

Secondary

A client with severe hypertension states, "I feel fine; I'm not really sick at all." The nurse will teach the client that the system/organs particularly targeted for damage by severe hypertension include which of the following?

Sensory

A nurse is caring for a client with acute mitral regurgitation related to an acute myocardial infarction. The nurse knows to monitor the client carefully for symptoms of which initial complication or result?

Severe heart failure Explanation: Acute mitral regurgitation usually manifests as severe congestive heart failure, resulting from blood flowing backward from the left ventricle to the left atria and eventually into the lungs. Kidney failure could become a problem later if cardiac output is too low, but not initially. CVA and an infarcted bowel would not be caused by mitral regurgitation.

A nurse is caring for a client who had a three-vessel coronary bypass graft 4 days earlier. His cholesterol profile is as follows: total cholesterol 265 mg/dl, low-density lipoprotein (LDL) 139 mg/dl, and high-density lipoprotein (HDL) 32 mg/dl. The client asks the nurse how to lower his cholesterol. The nurse should tell the client that:

She'll ask the dietitian to talk with him about modifying his diet.

A patient complains to the nurse about chest pain and palpitations during and after his morning jogs. The patient's family history reveals a history of coronary artery disease (CAD). What should the nurse recommend to minimize cardiac risk?

Smoking cessation Rationale: The first line of defense for patients with CAD is a change in lifestyle, such as smoking cessation, weight loss, stress management, and exercise. A protein-rich diet, liquid diet, and mild meals will not minimize cardiac risk.

The nurse assessing a patient with pericardial effusion at 0800 notes the apical pulse is 74 and the BP is 140/92. At 1000, the patient has neck vein distention, the apical pulse is 72, and the BP is 108/92. Which action would the nurse implement first?

Stay with the patient, use a calm voice, and ask for assistance via call light. Explanation: The nurse stays with the patient and continues to assess and record signs and symptoms while intervening to decrease patient anxiety. The pulse pressure is narrowing, and the patient is experiencing neck vein distention, indicative of rising central venous pressure. After reaching assistance via the call light from the patient's beside, the nurse notifies the physician immediately and prepares to assist with diagnostic echocardiography and pericardiocentesis. A left lateral recumbent position is used when administering enemas. Morphine would be given to someone who may be experiencing a myocardial infarction, not cardiac tamponade.

A 65-year-old male client with CAD has been prescribed a transdermal nitroglycerin patch. The nurse's instructions to the client would include which of the following? Select all that apply.

Store the patch in its original container when not in use. Remove the transdermal patch at night and reapply in the morning. Transdermal nitroglycerin systems are applied to the skin and slowly release nitroglycerin. Clients should be instructed to store the patch in its original container when not in use and keep tightly closed, remove the patch each night and reapply in the morning to prevent diminishing vasodilating effects, and expect possible side effects, such as headache, flushing, or nausea

Atrial fibrillation is common in patients with atrial septal defects (ASDs) and further increases the risk of which of the following?

Stroke Explanation: Atrial fibrillation is common in patients with ASDs and further increases the risk of stroke. Anticoagulation with aspirin is often prescribed. Cardiomegaly, heart failure, and splenomegaly may occur with infective endocarditis.

A client was transferring a load of fire wood from his front driveway to his backyard woodpile at 10 a.m. when he experienced a heaviness in his chest and dyspnea. He stopped working and rested, and the pain subsided. At noon, the pain returned. At 1:30 p.m., his wife took him to the emergency department. Around 2 p.m., the emergency department physician diagnoses an anterior myocardial infarction (MI). The nurse should anticipate which orders by the physician?

Sublingual nitroglycerin, tissue plasminogen activator (tPA), and telemetry The nurse should anticipate an order for sublingual nitroglycerin, tPA, and telemetry. (The client's chest pain began 4 hours before diagnosis.) The preferred choice is tPA, which is more specific for cardiac tissue than streptokinase. Stress testing shouldn't be performed during an MI. The client doesn't exhibit symptoms that indicate the use of lidocaine.

The nurse is educating the patient about administering nitroglycerin prior to discharge from the hospital. What information should the nurse include in the instructions?

Take a nitroglycerin and repeat every 5 minutes if the pain is not relieved until a total of 3 are taken. If pain is not relieved, activate the emergency medical system. Explanation: The nurse should recommend that the patient note how long it takes for the nitroglycerin to relieve the discomfort. Advise the patient that if pain persists after taking three sublingual tablets at 5-minute intervals, emergency medical services should be called.

The nurse is administering nitroglycerin, which he knows decreases preload as well as afterload. Preload refers to which of the following?

The amount of blood presented to the ventricles just before systole

A nurse is evaluating a client who had a myocardial infarction (MI) 7 days earlier. Which outcome indicates that the client is responding favorably to therapy?

The client demonstrates ability to tolerate more activity without chest pain. Explanation: The ability to tolerate more activity without chest pain indicates a favorable response to therapy in a client who is recovering from an MI or who has a history of coronary artery disease. The client should have a normal electrocardiogram with no arrhythmias and a regular heart rate of 60 to 100 beats/minute. Smoking is a cardiovascular risk factor that the client must be willing to eliminate. A client who responds favorably to therapy shouldn't have chest pain

A nurse is administering lanoxin, which she knows increases contractility as well as cardiac output. Contractility refers to which of the following?

The force of the contraction related to the status of the myocardium

A nurse is caring for a 30-year-old client diagnosed with atrial fibrillation who has just had a mitral valve replacement. The client is being discharged with prescribed warfarin (Comaudin). She mentions to you that she relies on the rhythm method for birth control. What education would be a priority for the nurse to provide to this client?

The high risk for complications if she becomes pregnant while taking warfarin

Two female nursing assistants approach a nurse on a cardiac step-down unit to report that a client who experienced an acute myocardial infarction made sexual comments to them. How should the nurse intervene?

The nurse should explain that the client might have concerns about resuming sexual activity but is afraid to ask. Sometimes clients are concerned about resuming sexual activity but are afraid to ask. Making inappropriate sexual comments provides a forum for asking questions. It isn't necessary to report the incident to the nursing supervisor immediately without investigating the situation further. The client's call light must be answered in a timely fashion. More information is needed before assuming that the client is asking for extra attention.

A patient with coronary artery disease (CAD) is having a cardiac catheterization. What indicator is present for the patient to have a coronary artery bypass graft (CABG)?

The patient has at least a 70% occlusion of a major coronary artery. For a patient to be considered for CABG, the coronary arteries to be bypassed must have approximately a 70% occlusion (60% if in the left main coronary artery).

A patient is being seen in a clinic to rule out mitral valve stenosis. Which assessment data would be most significant?

The patient reports shortness of breath when walking. Explanation: Dyspnea on exertion is typically the earliest manifestation of mitral valve stenosis. Late signs of right-sided heart failure are jugular vein distention, edema, and enlarged liver. Chest pain rarely occurs with mitral valve stenosis.

What is the hematocrit a measure of?

The proportion of blood volume that is occupied by red blood cell

A nurse is preparing a teaching plan regarding biological tissue valve replacement. Which of the following identifies a disadvantage of this type of valve replacement?

The valve has to be replaced frequently. Explanation: Biological valves deteriorate and need to be replaced frequently. They do not necessitate accompanying anticoagulant therapy. Infections are easier to treat and the risk of thromboembolism is lower as compared with mechanical valves.

In the treatment of coronary artery disease (CAD), medications are often ordered to control blood pressure in the client. Which of the following is a primary purpose of using beta-adrenergic blockers in the nursing management of CAD?

To decrease workload of the heart Beta-adrenergic blockers are used in the treatment of CAD to decrease the myocardial oxygen by reducing heart rate and workload of the heart. Nitrates are used for vasodilation. Anti-lipid drugs (such as statins and Bvitamins) are used to decrease homocysteine levels. ACE inhibitors inhibit the conversion of angiotensin.

A client in a clinic setting has just been diagnosed with hypertension. She asks what the end goal is for treatment. The correct reply from the nurse is which of the following?

To prevent complications/death by achieving and maintaining a blood pressure of 140/90 or less

A nurse is educating a community group about coronary artery disease. One member asks about how to avoid coronary artery disease. Which of the following items are considered modifiable risk factors for coronary artery disease? Choose all that apply.

Tobacco use Obesity Hyperlipidemia Rationale: Modifiable risk factors for coronary artery disease include hyperlipidemia, tobacco use, hypertension, diabetes mellitus, metabolic syndrome, obesity, and physical inactivity. Nonmodifiable risk factors include family history, advanced age, gender, and race.

Mary Seaver is admitted to the pediatrics unit where you practice nursing for treatment of rheumatic carditis. Aggressive antibiotic therapy and comfort measures have been instituted to minimize the long-lasting effects of the systemic inflammation. What is the best treatment for Mary's rheumatic carditis?

Treatment depends upon extent of cardiac involvement. Explanation: Concurrent treatment of rheumatic carditis depends on the extent of heart involvement. If minor, no treatment may be given; if heart failure or life-threatening dysrhythmias occur, extensive treatment is necessary. Surgery may be required to treat constrictive pericarditis and damage to heart valves. Prophylactic antibiotic therapy is recommended before future procedures that are associated with the dissemination of microorganisms that can lead to bacteremia and recurrence of endocarditis.

What is the most accurate serum determinant of an MI?

Troponin

The patient has had biomarkers drawn after complaining of chest pain. Which diagnostic of myocardial infarction remains elevated for as long as 2 weeks?

Troponin Rationale: Troponin remains elevated for a long period, often as long as 2 weeks, and it therefore can be used to detect recent myocardial damage. Myoglobin returns to normal in 12 hours. Total CK returns to normal in 3 days. CK-MB returns to normal in 3 to 4 days.

Which of the following is also termed preinfarction angina?

Unstable Angina Preinfarction angina is also known as unstable angina. Stable angina has predictable and consistent pain that occurs on exertion and it relieved by rest. Variant angina is exhibited by pain at rest with reversible ST-segment elevation. In silent angina, there is evidence of ischemia, but the patient reports no symptoms.

A nurse is teaching a client about maintaining a healthy heart. The nurse should include which point in her teaching?

Use alcohol in moderation The nurse should advise the client that alcohol may be used in moderation as long as there are no other contraindications for its use. Smoking, a diet high in cholesterol and saturated fat, and a sedentary lifestyle are all known risk factors for cardiac disease. The client should be encouraged to quit smoking, exercise three to four times per week, and consume a diet low in cholesterol and saturated fat.

The nurse knows that women and the elderly are at greater risk for a fatal myocardial event. Which factor is the primary contributor of this cause?

Vague symptoms Often, women and elderly do not have the typical chest pain associated with a myocardial infarction. Some report vague symptoms (fatigue, abdominal pain), which can lead to misdiagnosis. Some older adults may experience little or no chest pain. Gender is not a contributing factor for fatal occurrence but rather a result of symptoms association.

A patient, who is resting quietly in a step-down cardiac care unit, reports chest pain. The cardiac monitor indicates the presence of reversible ST-segment elevation. The nurse understands that the patient may be experiencing coronary artery vasospasm. This is a type of angina known as:

Variant Explanation: Variant or Prinzmetal's angina is distinguished by its occurrence during rest, as opposed to stable angina, which occurs with activity. Silent angina occurs without symptoms, and intractable angina is evidenced by incapacitating pain.

The nurse understands that which of the following medications will be administered for 6 to 12 weeks following prosthetic porcine valve surgery?

Warfarin Explanation: To reduce the risk of thrombosis in patients with porcine or bovine tissue valves, warfarin is required for 6 to 12 weeks, followed by aspirin therapy. Furosemide would not be given for 6 to 12 weeks following this type of surgery. Digoxin may be used for the treatment of arrhythmias, but not just for 6 to 12 weeks.

A 23-year-old female client has been diagnosed with Raynaud's disease. The nurse teaches the client which of the following self-care strategies to minimize risks associated with this disease? Select all that apply.

Wear gloves to protect hands from injury when performing tasks. Do not smoke or stop smoking. Avoid over-the-counter decongestants and cold remedies The nurse instructs clients with Raynaud's disease to quit smoking, avoid over-the-counter decongestants, cold remedies, and drugs for symptomatic relief of hay fever because of their vasoconstrictive qualities, protect hands and feet from injury, and wear warm socks and mittens when going outdoors in the cold weather

Which nursing actions would be of greatest importance in the management of a client preparing for angioplasty?

Withhold anticoagulant therapy. The nurse knows to withhold the anticoagulant therapy to decrease chance of hemorrhage during the procedure. The nurse does inform the client of diagnostic test, will assess pulses, and prep the skin prior to the angioplasty, but this is not the most important action to be taken.

A client with chest pain doesn't respond to nitroglycerin. When he's admitted to the emergency department, the health care team obtains an electrocardiogram and administers I.V. morphine. The physician also considers administering alteplase (Activase). This thrombolytic agent must be administered how soon after onset of myocardial infarction (MI) symptoms?

Within 6 hours For the best chance of salvaging the client's myocardium, a thrombolytic agent must be administered within 6 hours after onset of chest pain or other signs or symptoms of MI. Sudden death is most likely to occur within the first 24 hours after an MI. Physicians initiate I.V. heparin therapy after administration of a thrombolytic agent; it usually continues for 5 to 7 days

Patients who are taking beta-adrenergic blocking agents should be cautioned not to stop taking their medications abruptly because which of the following may occur?

Worsening angina Patients taking beta blockers are cautioned not to stop taking them abruptly because angina may worsen and myocardial infarction may develop. Beta blockers do not cause the formation of blood clots, internal bleeding, or thrombocytopenia.

Which type of graft is utilized when a heart valve replacement is made of tissue from an animal heart valve?

Xenograft

During the insertion of a rigid scope for bronchoscopy, a client experiences a vasovagal response. The nurse should expect:

a drop in the client's heart rate. Explanation: During a bronchoscopy, a vasovagal response may be caused by stimulating the pharynx, and it, in turn, may cause stimulation of the vagus nerve. The client may, therefore, experience a sudden drop in heart rate leading to syncope. Stimulation of the vagus nerve doesn't lead to pupillary dilation or bronchodilation. Stimulation of the vagus nerve increases gastric secretions.

An obese white male client, age 49, is diagnosed with hypercholesterolemia. The physician orders a low-fat, low-cholesterol, low-calorie diet to reduce blood lipid levels and promote weight loss. This diet is crucial to the client's well-being because his race, sex, and age increase his risk for coronary artery disease (CAD). To determine whether the client has other major risk factors for CAD, the nurse should assess for:

a history of diabetes mellitus. Explanation: Diabetes mellitus, smoking, and hypertension are other major risk factors for CAD. Elevated HDL levels aren't a risk factor for CAD; in fact, increased HDL levels seem to protect against CAD. Ischemic heart disease is another term for CAD, not a risk factor. Alcoholism hasn't been identified as a major risk factor for CAD

The nurse is caring for patient experiencing an acute MI (STEMI). The nurse anticipates the physician will prescribe alteplase (Activase). Prior to administering this medication, which of the following questions is most important for the nurse to ask the patient?

a) "What time did your chest pain start today?" Explanation: The patient may be a candidate for thrombolytic (fibrolytic) therapy. These medications are administered if the patient's chest pain lasts longer than 20 minutes, unrelieved by nitroglycerin, ST-segment elevation in the at least two leads that face the same area of the heart, less than 6 hours from onset of pain. The most appropriate question for the nurse to ask is in relationship to when the chest pain began. The other questions would not aid in determining if the patient is a candidate for thrombolytic therapy.

A patient has had a 12-lead -ECG completed as part of an annual physical examination. The nurse notes an abnormal Q wave on an otherwise unremarkable ECG. The nurse recognizes this finding indicates which of the following?

a) A past MI Explanation: An abnormal Q wave may be present without ST-segment and T-wave changes, which indicates an old, not acute, MI.

A patient diagnosed with a myocardial infarction (MI) has begun an active rehabilitation program. The nurse recognizes an overall goal of rehabilitation for a patient who has had an MI includes which of the following?

a) Improvement of the quality of life Explanation: Overall, cardiac rehabilitation is a complete program dedicated to extending and improving quality of life. An immediate objective of rehabilitation of the MI patient is to limit the effects and progression of atherosclerosis. An immediate objective of rehabilitation of the MI patient is to return the patient to work and a preillness lifestyle. An immediate objective of rehabilitation of the MI patient is to prevent another cardiac event.

Which assessment finding by the nurse is the most significant finding suggestive of aortic aneurysm?

abdomen bruit

A client has had a 12-lead ECG completed as part of an annual physical examination. The nurse notes an abnormal Q wave on an otherwise unremarkable ECG. The nurse recognizes that this finding indicates

an old MI

A client reports pain and cramping in the thigh when climbing stairs and numbness in the legs after exertion. Which diagnostic test with the physician likely perform right in the office to determine PAD?

ankle-brachial index

A client has a blockage in the proximal portion of a coronary artery. After learning about treatment options, the client decides to undergo percutaneous transluminal coronary angioplasty (PTCA). During this procedure, the nurse expects to administer an:

anticoagulant. Rationale: During PTCA, the client receives heparin, an anticoagulant, as well as calcium agonists, nitrates, or both, to reduce coronary artery spasm. Nurses don't routinely give antibiotics during this procedure; however, because the procedure is invasive, the client may receive prophylactic antibiotics to reduce the risk of infection. An antihypertensive may cause hypotension, which should be avoided during the procedure. An anticonvulsant isn't indicated because this procedure doesn't increase the risk of seizures.

A client with chronic arterial occlusive disease undergoes percutaneous transluminal coronary angioplasty (PTCA) for mechanical dilation of the right femoral artery. After the procedure, the client will require long-term administration of:

aspirin or clopidogrel (Plavix). fter PTCA, the client begins long-term aspirin or clopidogrel therapy to prevent thromboembolism. Physicians order heparin for anticoagulation during this procedure; some physicians discharge clients with a prescription for long-term warfarin (Coumadin) or low-molecular-weight heparin therapy. Pentoxifylline, a vasodilator used to treat chronic arterial occlusion, isn't required after PTCA because the procedure itself opens the vessel. The physician may order short-term acetaminophen therapy to manage fever or discomfort, but prolonged therapy isn't warranted. The client may need an antibiotic, such as penicillin or erythromycin, for a brief period to prevent infection associated with an invasive procedure; long-term therapy isn't necessary.

A client reports chest pain and heavy breathing when exercising or when stressed. Which is a priority nursing intervention for the client diagnosed with coronary artery disease?

assess chest pain and administer prescribed drugs and oxygen

An older adult is postoperative day one, following a coronary artery bypass graft (CABG).The client's family members express concern to the nurse that the client is uncharacteristically confused and does not know where he is. After reporting these change in status to the care provider, the nurse should:

assess for factors that may be causing the client's delirium

A nurse is caring for a client receiving warfarin (Coumadin) therapy following a mechanical valve replacement. The nurse completed the client's prothrombin time and International Normalized Ratio (INR) at 7 a.m., before the morning meal. The client had an INR reading of 4. The nurse's first priority should be to:

assess the client for bleeding around the gums or in the stool and notify the physician of the laboratory results and most recent administration of warfarin. Explanation: For a client taking warfarin following a valve replacement, the INR should be between 2 and 3.5. The nurse should notify the physician of an elevated INR level and communicate assessment data regarding possible bleeding. The nurse shouldn't administer medication such as warfarin or vitamin K without a physician's order. The nurse should notify the physician before holding a medication scheduled to be administered during another shift.

The nurse is presenting a workshop at the senior citizens center about how the changes of aging predispose clients to vascular occlusive disorders. What would the nurse name as the most common cause of peripheral arterial problems in the older adult?

atherosclerosis

In order to be effective, percutaneous transluminal coronary angioplasty (PTCA) must be performed within what time frame, beginning with arrival at the emergency department after diagnosis of myocardial infarction (MI)?

b) 60 minutes Explanation: The 60-minute interval is known as "door-to-balloon time" for performance of PTCA on a diagnosed MI patient. The 30-minute interval is known as "door-to-needle time" for administration of thrombolytics post MI. The time frame of 9 days refers to the time for onset of vasculitis after administration of streptokinase for thrombolysis in an acute MI patient. The 6- to 12-month time frame refers to the time period during which streptokinase will not be used again in the same patient for acute MI.

Following a percutaneous coronary intervention (PCI), a patient is returned to the nursing unit with large peripheral vascular access sheaths in place. The nurse understands that which of the following methods to induce hemostasis after sheath is contraindicated?

b) Application of a sandbag to the area Explanation: Applying a sandbag to the sheath insertion site is ineffective in reducing the incidence of bleeding and is not an acceptable standard of care. Application of a vascular closure device (Angioseal, VasoSeal), direct manual pressure to the sheath introduction site, and application of a mechanical compression device (C-shaped clamp) are all appropriate methods used to induce hemostasis following peripheral sheath removal.

The nurse understands it is important to promote adequate tissue perfusion following cardiac surgery. Which of the following measures should the nurse complete to prevent deep venous thrombosis (DVT) and possible pulmonary embolism (PE) development? Select all that apply.

b) Apply antiembolism stockings., d) Initiate passive exercises., e) Avoid elevating the knees on the bed. Explanation: Preventative measures utilized to prevent venous stasis include: Application of sequential pneumatic compression wraps or antiembolic stockings; discouraging leg crossing; avoiding elevating the knees on the bed; omitting pillows in the popliteal space; beginning passive exercises followed by active exercises to promote circulation and prevent venous stasis.

The nurse is caring for a patient diagnosed with unstable angina receiving IV heparin. The patient is placed on bleeding precautions. Bleeding precautions include which of the following measures?

b) Avoiding continuous BP monitoring Explanation: The patient receiving heparin is placed on bleeding precautions, which can include: applying pressure to the site of any needle punctures for a longer time than usual, avoiding intramuscular injections, avoiding tissue injury and bruising from trauma or constrictive devices (e.g. continuous use of an automatic BP cuff). SQ injections are permitted; a soft toothbrush should be used, and the patient may use nail clippers, but with caution.

The nurse is caring for a patient newly diagnosed with coronary artery disease (CAD). While developing a teaching plan for the patient to address modifiable risk factors for CAD, the nurse will include which of the following? Select all that apply.

b) Elevated blood pressure, c) Obesity Explanation: Hypertension, obesity, hyperlipidemia, tobacco use, diabetes mellitus, metabolic syndrome, and physical inactivity are modifiable risk factor for CAD. Alcohol and drug use are not included in the list of modifiable risk factors for CAD.

The nurse is reviewing the laboratory results for a patient diagnosed with coronary artery disease (CAD). The patient's low-density lipoprotein (LDL) level is 115 mg/dL. The nurse interprets this value as which of the following?

b) High Explanation: The normal LDL range is 100 mg/dL to 130 mg/dL. A level of 115 mg/dL is considered to be high. The goal of treatment is to decrease the LDL level below 100 mg/dL (less than 70 mg/dL for very high-risk patients).

The nurse is caring for a patient presenting to the emergency department (ED) complaining of chest pain. Which of the following electrocardiographic (ECG) findings would be most concerning to the nurse?

b) ST elevations Explanation: The first signs of an acute MI are usually seen in the T wave and ST segment. The T wave becomes inverted; the ST segment elevates (usually flat). An elevation in ST segment in two contiguous leads is a key diagnostic indicator for MI (i.e. ST elevation myocardial infarction, STEMI). This patient requires immediate invasive therapy or fibrinolytic medications. Although the other ECG findings require intervention, elevated ST elevations require immediate and definitive interventions.

When a patient who has been diagnosed with angina pectoris complains that he is experiencing chest pain more frequently even at rest, the period of pain is longer, and it takes less stress for the pain to occur, the nurse recognizes that the patient is describing which type of angina?

b) Unstable Explanation: Unstable angina is also called crescendo or preinfarction angina and indicates the need for a change in treatment. Intractable or refractory angina produces severe, incapacitating chest pain that does not respond to conventional treatment. Variant angina is described as pain at rest with reversible ST-segment elevation and is thought to be caused by coronary artery vasospasm. Intractable or refractory angina produces severe, incapacitating chest pain that does not respond to conventional treatment.

The nurse is reevaluating a patient 2 hours following a percutaneous transluminal coronary angioplasty (PTCA) procedure. Which of the following assessment findings may indicate the patient is experiencing a complication of the procedure?

b) Urine output of 40 mL Explanation: Complications that may occur following a PTCA include myocardial ischemia, bleeding and hematoma formation, retroperitoneal hematoma, arterial occlusion, pseudoaneurysm formation, arteriovenous fistula formation, and acute renal failure. The urine output of 40 mL over a 2-hour period may indicate acute renal failure. The patient is expected to have a minimum urine output of 30 mL per hour. Dried blood at the insertion site is a finding warranting no acute intervention. A serum potassium level of 4.0 mEq/L is within normal range. The heart rate of 100 bmp is within the normal range and indicates no acute distress.

A patient presents to the emergency room complaining of chest pain. The patient's orders include the following elements. Which order should the nurse complete first?

c) 12-lead ECG Explanation: The nurse should complete the 12-lead ECG first. The priority is to determine if the patient is suffering an acute MI and implement appropriate interventions as quickly as possible. The other orders should be completed after the ECG.

The nurse is caring for a patient who was admitted to the telemetry unit with a diagnosis of rule/out acute MI. The patient's chest pain began 3 hours ago. Which of the following laboratory tests would be most helpful in confirming the diagnosis of a current MI?

c) Creatinine kinase-myoglobin (CK-MB) level Explanation: Elevated CK-MB assessment by mass assay is an indicator of acute MI; the levels begin to increase within a few hours and peak within 24 hours of an MI. If the area is reperfused (due to thrombotic therapy or PCI), it peaks earlier. CK-MM (skeletal muscle) is not an indicator of cardiac muscle damage. There are three isomers of troponin: C, I, and T. Troponin I and T are specific for cardiac muscle, and these biomarkers are currently recognized as reliable and critical markers of myocardial injury. An increase in myoglobin is not very specific in indicating an acute cardiac event; however, negative results are an excellent parameter for ruling out an acute MI.

A patient admitted to the coronary care unit (CCU) diagnosed with a STEMI is anxious and fearful. Which of the following medications will the nurse administer to relieve the patient's anxiety and decrease cardiac workload?

c) IV morphine Explanation: IV morphine is the analgesic of choice for treatment of an acute MI. It is given to reduce pain and treat anxiety. It also reduces preload and afterload, which decreases the workload of the heart. IV nitroglycerin is given to alleviate chest pain. Administration of Tenormin and Norvasc are not indicated in this situation.

The nurse recognizes that the treatment for a non-ST elevation myocardial infarction (NSTEMI) differs from that of a patient with a STEMI, in that a STEMI is more frequently treated with which of the following?

c) Percutaneous coronary intervention (PCI) Explanation: The patient with a STEMI is often taken directly to the cardiac catheterization laboratory for an immediate PCI. Superior outcomes have been reported with the use of PCI compared to thrombolytics. IV heparin and IV nitroglycerin are used to treat NSTEMI.

A client with CAD thinks diltiazem (Cardizem) has been causing nausea. Diltiazem (Cardizem) is categorized as which type of drug?

calcium-channel blocker

Which complication of cardiac surgery occurs when fluid and clots accumulate in the pericardial sac, which compresses the heart, preventing blood from filling the ventricles?

cardiac tamponade

A nurse is caring for a client in the cardiovascular intensive care unit following a coronary artery bypass graft. Which clinical finding requires immediate intervention by the nurse?

central venous pressure reading of 1

A middle-aged client presents to the ED reporting severe chest discomfort. Which finding is most indicative of a possible myocardial infarction (MI)?

chest discomfort not relieved by rest or nitroglycerin

A client is admitted for treatment of Prinzmetal's angina. When developing this client's care plan, the nurse should keep in mind that this type of angina can result from:

coronary artery spasm. Explanation: Prinzmetal's angina results from coronary artery spasm. Activities that increase myocardial oxygen demand may trigger angina of effort. An unpredictable amount of activity may precipitate unstable angina. Worsening angina is brought on by the same type or level of activity that caused previous angina episodes; anginal pain becomes increasingly severe.

The nurse has completed a teaching session on the self-administration of sublingual nitroglycerin. Which of the following patient statements indicates that the patient teaching has been effective?

d) "I can take nitroglycerin prior to having sexual intercourse so I won't develop chest pain". Explanation: Nitroglycerin can be taken in anticipation of any activity that may produce pain. Because nitroglycerin increases tolerance for exercise and stress when taken prophylactically (i.e. before angina-producing activity, such as exercise, stair-climbing, or sexual intercourse), it is best taken before pain develops. The client is instructed to take three tablets 5 minutes apart and if the chest pain is not relieved emergency medical services should be contacted. Nitroglycerin is very unstable; it should be carried securely in its original container (e.g., capped dark glass bottle); tablets should never be removed and stored in metal or plastic pillboxes. Side effects of nitroglycerin includes: flushing, throbbing headache, hypotension, and tachycardia.

A client presents to the emergency room with characteristics of atherosclerosis. What characteristics would the client display?

fatty deposits in the lumen of arteries

What is a characteristic of atherosclerosis?

fatty deposits in the lumen of arteries

A nurse is teaching a client how to take nitroglycerin to treat angina pectoris. The client verbalizes an understanding of the need to take up to three sublingual nitroglycerin (Nitrostat) tablets at 5-minute intervals, if necessary, and to notify the physician immediately if chest pain doesn't subside within 15 minutes. The nurse tells the client that, after taking the nitroglycerin, he may experience:

headache, hypotension, dizziness, and flushing. Explanation: Headache, hypotension, dizziness, and flushing are classic adverse effects of nitroglycerin, a vasodilator. Vasodilators, beta-adrenergic blockers, and calcium channel blockers are three major classes of drugs used to treat angina pectoris. Nausea, vomiting, depression, fatigue, and impotence are adverse effects of propranolol, a beta-adrenergic blocker. Sedation, nausea, vomiting, constipation, and respiratory depression are common adverse effects of morphine, an opioid analgesic that relieves pain associated with acute myocardial infarction. Flushing, dizziness, headache, and pedal edema are common adverse effects of nifedipine, a calcium channel blocker.

The laboratory values for a client diagnosed with coronary artery disease (CAD) have just come back from the lab. The client's low-density lipoprotein (LDL) level is 112 mg/dL. This nurses recognizes that this value is

high

A client's lipid profile reveals an LDL level of 122 mg/dL. This is considered a:

high LDL level

When the postcardiac surgery client demonstrates restlessness, nausea, weakness, and peaked T waves, the nurse reviews the client's serum electrolytes, anticipating which abnormality?

hyperkalemia

A patient who had CABG is exhibiting signs of cardiac failure. What medications does the nurse anticipate administering for this patient? Select all that apply:

inotropic agents diuretics digoxin

Which are consistent with manifestations of metabolic syndrome?

insulin resistance

Which s the analgesic of choice for acute myocardial infarction (MI)?

morphine The analgesic of choice for acute MI is morphine administered in IV boluses to reduce pain and anxiety. Aspirin is an antiplatelet medication. Meperidine and Ibuprofen are not the analgesics of choice.

Creatine kinase-MB isoenzyme (CK-MB) can increase as a result of:

myocardial necrosis. Explanation: An increase in CK-MB is related to myocardial necrosis. An increase in total CK might occur for several reasons, including brain injury, such as cerebral bleeding; skeletal muscle damage, which can result from I.M. injections or falls; muscular or neuromuscular disease; vigorous exercise; trauma; or surgery.

After 2-hour onset of acute chest pain, the client is brought to the emergency department for evaluation. Elevation of which diagnostic findings would the nurse identify as suggestive of an acute myocardial infarction at this time?

myoglobin Myoglobin is a biomarker that rises in 2 to 3 hours after heart damage. Troponin is the gold standard for determining heart damage, but troponin I levels due not rise until 4 to 6 hours after MI. WBCs and C-reactive protein levels will rise but not until about day 3.

The nurse is caring for a client who is being evaluated for lipid-lowering medication. The client's laboratory results reveal the following: total cholesterol 230 mg/dL, LDL 120 mg/dL, triglyceride level 310 mg/dL. Which class of medications would be most appropriate for the client based on these laboratory findings?

nicotinic acid

The nurse recognizes that the treatment for a non-ST-elevation myocardial infarction (NSTEMI) differs from that for a STEMI, in that a STEMI is more frequently treated with

percutaneous coronary intervention (PCI)

A client presents to the ED with a myocardial infarction. Prior to administering a prescribed thrombolytic agent, the nurse must determine whether the client has which absolute contraindication to thrombolytic therapy?

prior intracranial hemorrhage Explanation: History of a prior intracranial hemorrhage is an absolute contraindication for thrombolytic therapy. An allergy to iodine, shellfish, radiographic dye, and latex are of primary concern before a cardiac catheterization but not a known contraindication for thrombolytic therapy. Administration of a thrombolytic agent with heparin increases risk of bleeding; the primary healthcare provider usually discontinues the heparin until thrombolytic treatment is completed.

The nurse is caring for a client following a coronary artery bypass graft (CABG). The nurse notes persistent oozing of bloody drainage from various puncture sites. The nurse anticipates that the physician will order which medication to neutralize the unfractionated heparin the client received?

protamine sulfate Protamine sulfate is known as the antagonist for unfractionated heparin (it neutralizes heparin). Alteplase is a thrombolytic agent. Clopidogrel is an antiplatelet medication that is given to reduce the risk of thrombus formation after coronary stent placement. The antiplatelet effect of aspirin does not reverse the effects of heparin.

A client has had oral anticoagulation ordered. What should the nurse monitor for when the client is taking oral anticoagulation?

prothrombin time (PT) or international normalized ratio (INR)

When providing discharge instructions for a client who has been prescribed sublingual nitroglycerin for angina, the nurse should plan to include which instructions?

see if rest relieves the chest pain before using the nitroglycerin Decreased activity may relieve chest pain; sitting will prevent injury should the nitroglycerin lower BP and cause fainting. The client should expect to feel dizzy or flushed or to develop a headache following sublingual nitroglycerin use. The client should place one nitroglycerin tablet under the tongue if 2-3 minutes of rest fails to relieve pain. Clients may take up to three nitroglycerin tablets within 5 minutes of each other to relieve angina. However, they should call 911 if the three tablets fail to resolve the chest pain.

A patient with infective endocarditis of a prosthetic mitral valve returns to the emergency department with a second episode of left-sided weakness and visual changes. The nurse expects that collaborative management of the patient will include

surgical valve replacement Explanation: Aortic or mitral valve debridement, excision, or replacement is required in patients with more than one serious systemic embolic episode.

A client with severe angina pectoris and ST-segment elevation on an electrocardiogram is being seen in the emergency department. In terms of diagnostic laboratory testing, it's most important for the nurse to advocate ordering a:

troponin level. Rationale: Troponin is a myocardial cell protein that is elevated in the serum when myocardial damage has occurred during a myocardial infarction (MI). It's the best serum indicator of MI and is more indicative of cardiac damage than creatine kinase. Hb values and liver panel components aren't as useful in the diagnosis of MI as a troponin level.

A client with severe angina pectoris and electrocardiogram changes is seen by a physician in the emergency department. In terms of serum testing, it's most important for the physician to order cardiac:

troponin. This client exhibits signs of myocardial infarction (MI), and the most accurate serum determinant of an MI is troponin level. Creatine kinase, lactate dehydrogenase and myoglobin tests can show evidence of muscle injury, but they're less specific indicators of myocardial damage than troponin.

Which of the following medications could be used to treat pericarditis? Choose all that apply.

• Colchicine • Motrin • Prednisone Explanation: Analgesics and nonsteroidal anti-inflammatory drugs (NSAIDs), such as aspirin or ibuprofen (Motrin), may be prescribed for pain relief during the acute phase. Indomethacin (Indocin) is contraindicated, because it may decrease coronary blood flow. Corticosteroids (eg, prednisone) may be prescribed if the pericarditis is severe or the patient does not respond to NSAIDs. Colchicine may also be used as alternative therapy.

Which of the following lab values would be seen in the patient diagnosed with infective endocarditis? Select all that apply.

• Elevated erythrocyte sedimentation rate (ESR) • Elevated white blood cell (WBC) count • Elevated c-reactive protein Explanation: Abnormal findings include anemia, elevated WBC counts, elevated ESR, and elevated c-reactive protein

The nurse has been asked to teach a patient how to self-administer nitroglycerin. The nurse should instruct the patient to do which of the following? Select all of the teaching points that apply.

• Let the tablet dissolve in the mouth and keep the tongue still. The tablet can be crushed between the teeth but not swallowed. • Renew the supply every 6 months. • Take the tablet in anticipation of any activity that can produce pain. • Call emergency services if, after taking three tablets (one every 5 minutes), pain persists. Rationale: Nitroglycerine is very unstable and should be carried securely in its original container (capped, dark, glass bottle). The tablets should never be removed and stored in metal or plastic pillboxes. Nitroglycerine is also volatile and is inactivated by heat, moisture, air, light, and time. Therefore, storage and replacement is recommended every 6 months. Refer to Box 14-3 in the text.

A nurse is teaching nitroglycerin to a client with hospitalized client with coronary artery disease who is being discharged. The nurse tells the client that nitroglycerin has which of the following actions? Choose all that apply.

• Reduces myocardial oxygen consumption • Dilates blood vessels • Decreases ischemia • Relieves pain Explanation: Nitroglycerin dilates blood vessels and reduces the amount of blood returning to the heart, which reduces the workload of the heart and myocardial oxygen consumption. As the dilated vessels allow more blood supply to the heart, ischemia and pain are reduced. Nitroglycerin does not affect the urge to use tobacco.

Which of the following are the first symptoms of cardiac tamponade? Select all that apply.

• Shortness of breath • Chest tightness • Dizziness Explanation: The first symptoms of cardiac tamponade are often shortness of breath, chest tightness, dizziness, or restlessness. The patient may have tachycardia. Neck vein distention and other signs of rising central venous pressure develop.


Ensembles d'études connexes

(Handout) Practice Multiple Choice for Demand and Supply

View Set

Saunders | Pediatric: Integumentary

View Set

Texas Real Estate Missed Questions

View Set

ALAT-ALAT OPTIK (Chandra Kirana)

View Set

CompTIA Security+ SY0-501 Part 1 (316 Questions)

View Set